Что такое втягивающее реле: Втягивающее реле стартера — неисправности, проверка, принцип, схема работы и ремонт

Содержание

основные неисправности и особенности выбора

В морозную погоду нередки ситуации, когда при попытках завести автомобиль можно услышать лишь шум работы всего стартера или же отдельных его компонентов. Конечно, замена стартера могла бы решить проблему, однако часто неисправность касается лишь двух его компонентов – бендикса, втягивающего реле. Именно о втором компоненте мы и поговорим, заодно ответив на вопросы примерно такого содержания: как проверить втягивающее реле, как его отремонтировать и как подбирать новое реле в случае необходимости.

Немного теории

Вообще, современные стартеры оснащаются парой реле. Первое отвечает за включение агрегата. Оно находится непосредственно в моторном отсеке, причем в отдельных моделях автомобилей реле помещается еще и в отдельный корпус. Второе реле, называемое тяговым (втягивающим), устанавливают прямо на стартер. Функций у данного устройства несколько:

  • Обеспечение синхронной работы узлов автомобильного стартера при запуске ДВС;
  • Распределение эл. энергии между эл-магнитным реле и моторчиком стартера;
  • Обеспечение совместной работы обгонной муфты и венца маховика (подвод шестерней) и, наоборот, отвод шестерней в нужный момент времени.

Вот как все устроено: когда в замке зажигания были замкнуты контакты, сразу же срабатывает обычное реле стартера , отвечающее за подачу напряжения от аккумулятора к т.н. втягивающую обмотку. В результате наводится магнитное поле, воздействующее на якорь реле, за счет тот попадает внутрь обмотки. Теперь осуществляется сразу 2 действия: начинает двигаться вилка стартера, помогая сдвинуться обгонной муфте, еще называемая бендиксом, а также замыкаются контакты установленного втягивающего реле. Проще говоря: стартер теперь соединен с маховиком, после чего он подключается к аккумулятору и за счет прохождения тока по цепи теперь может запуститься двигатель. Не забывайте, что работа систем сгорания без исправной работы электрических цепей авто невозможна.

Однако, и на этом еще не все. Теперь, когда стартер включается, за счет работы т.н. удерживающей катушки якорь удерживается в своем крайнем положении – в таком режиме работы втягивающее реле не потребляет много энергии. А если двигатель уже запущен, цепь стартера разрывается и происходит обесточивание реле. В автомобильных реле имеется пружина – именно она, стремясь сжаться, возвращает якорь в начальное положение. Следом свое положение меняет контактный диск и бендикс. Аккумулятор отключается. Далее реле уже никак не участвует в работе автомобиля вплоть до дальнейшего запуска двигателя.

Подробнее об устройстве

Наверняка не каждый автолюбитель сразу же поймет все нюансы работы автомобильных реле. Мы постараемся немного упростить и разобрать важные моменты. Важно понимать, что реле – это устройство, предназначенное для замыкания или размыкание электрической цепи, в которую оно введено. Конкретно тяговое реле реагирует на электрические величины и не работает на принципах электромагнитизма. Обратите внимание на изображение ниже.

В простейшем втягивающем реле имеет корпус, якорь, пружина, контакты и магнит с парой обмоток (катушек). Первая катушка, называемая втягивающей, соединена с электродвигателем и клеммой управление, теперь временем как вторая (удерживающая катушка) имеет связь с корпусом и выводом управления.

При подаче постоянного тока к контакту управления в катушке наводится магнитное поле, оказывающее действие на якорь реле. Тот начинает движение и далее обеспечивает замыкание цепи между стартером и аккумулятором. Когда питание не подается, пружина возвращает якорь в его начальное положение – контакты тут же размыкаются. Свою работу бендикс прекращает.

Основные неисправности

Втягивающее реле хоть и устроено довольно просто, может пострадать от множества внешних и внутренних воздействий. Первое и самое очевидное: разрушение материалов, из которых реле состоит. Вторая и даже более распространенная проблема: выход реле из строя вследствие сгорания контактных пластин (часто их называет пятаками). Третье: сгорание обмотки (тоже весьма частая проблема). Почему горит втягивающее реле стартера? Самая частая причина неисправностей подобного характера кроется в электрохимических процессах, наблюдающихся в реле при попадании воды.

К признакам неисправностей имеющегося в автомобиле втягивающего реле агрегата можно отнести следующее:

  • Стартер осуществляет работу даже после запуска ДВС. Выявить это можно по звуку;
  • Стартер работает вхолостую. При этом ДВС не запускается;
  • Стартер не начал работу после характерного щелчка, сигнализирующего о включении устройства.

Дабы снять подозрения со всех смежных с реле компонентов системы, проделайте следующее:

  • Замкнуть контактные болты на тыльной части стартера, пользуясь куском провода. Ток будет идти в обход втягивающее реле;
  • Провернуть ключ зажигания. Если стартер начал вращение, проблема точно не в нем, а в реле, силовые контакты которого подгорели;
  • Послушать, как работает реле. После проворачивания ключа оно должно щелкать. Если при этом стартер не включился в работу, само реле исправно.

Проще говоря, реле может работать, а сам стартер не вращается, или же и стартер, и реле не работают вовсе. Нередки ситуации, когда реле исправно, но при его работе слышен стук. Причина кроется в плохом контакте между обмотками и «массой». Величина сопротивления исправной втягивающей обмотки составляет 0,55 Ом, тем временем как удерживающей обмотки чуть больше – 0,75 Ом. Сопротивления нужно измерить омметром. Если реальные показатели сопротивлений ниже указанных, можно говорить о произошедшем коротком замыкании в обмотках. Высокий показатель сопротивления говорит о плохом контакте или с клеммами, или с «массой». Для таких проверок лучше не пользоваться лампочкой, так как она будет гореть даже при коротком замыкании.

Также нельзя не рассказать об одной интересной неисправности, которая нехарактерна для правильно изготовленных втягивающих реле. Дело в том, что вывод одной из катушек может быть осуществлен неправильно – при работе катушке начинают компенсировать друг друга. В этом случае нужно покупать новое реле, хотя можно попробовать перепаять концы дефективной катушки.

Как отремонтировать втягивающее реле стартера

Бывает и так, что реле удается отремонтировать. Заметьте, что далеко не все из этих устройств являются разборными. Если у вас разборное, то все отлично. Но для начала реле нужно снять. Работа состоит из таких этапов:

  1. Отключить аккумуляторную батарею;
  2. Снять весь стартер с автомобиля;
  3. Тщательнейшим образом очистить стартер – грязь и пыль в ходе работы может попасть внутрь агрегата;
  4. Открутить гайки щеточного узла, после чего ослабить болт контакта втягивающего реле;
  5. Окрутить винты, фиксирующие реле;
  6. Заняться осмотром демонтированного реле.

Если реле разборное, то на его торцах будут видны гайки – их нужно открутить и разобрать устройство. Заметьте, что крышка некоторых реле не снимается без предварительной распайки контактов. Имеет смысл вне зависимости от характера неисправности зачистить все токоведущие части (обратите внимание на цвет материалов в местах пригорания). Эксперты рекомендуют при любом ремонте менять сердечник реле на новый. Также может потребоваться замена контактов. Если возвратная пружина была деформирована или ослаблена, ее придется заменить. Если была обнаружена проблема с компенсирующимися катушками, описанная выше, потребуется пайка. При обратной сборке и установке реле уделите особое внимание силе затяжки клемм.

Эксперты не рекомендуют смазывать втягивающее реле стартера, а вот обгонную муфту (бендикс

) смазывать можно. При этом не рекомендуется использовать графитную смазку, так как она боится высоких температур, воды, а также довольно быстро высыхает.

Выбор нового втягивающего реле

Подбирать новое тяговое реле раньше было проще всего именно в офлайн-магазинах. Сегодня поиск в онлайн-магазинах серьезно упростился и стал проще. При подборе нового втягивающего реле нужно помнить о следующем:

  • Не всегда реле, которое вы найдете в электронных каталогах, соответствует стартеру. Искать стоит в первую очередь оригинальное реле для установленного в вашем авто стартере, а уже потом подбирать аналоги данного реле, если оно показалось вам слишком дорогим;
  • Уточните геометрию реле, которое будете заказывать. Вот здесь-то офлайн магазины и оказываются более надежными – можно взять старое реле с собой и сравнить его с найденным;
  • Стоит заранее приобрести 2 пары медных гайк и шайб, которыми лучше всего фиксировать новую деталь. Запас такой «мелочи» рано или поздно окажется крайне полезным.

Советуем отнестись к выбору втягивающего реле с особым вниманием. Даже у вас стоит оригинальный стартер, на аналогичной модели авто другого года выпуска стартер может оказаться другим – пара установленных на агрегатах реле не будет взаимозаменяемой. Именно по этой причине нужно руководствоваться или VIN-кодом, или параметрами и кодом имеющегося стартера и втягивающего реле.

Ведущие производители

При покупке втягивающего реле стартера важно обращать внимание не только на его характеристики и на совместимость с имеющимся стартером, но также и на фирму-производителя. Учтите, что автозапчасти могут предлагать не только сами производители, но также и фирмы-упаковщики. Вот несколько марок, о которых автолюбителю стоит знать:

Последняя фирма является не столько производителем, сколько упаковщиком. Именно ее автозапчасти водитель сможет найти в магазине с наибольшей вероятностью. Качество можно оценить как высокое, а цену как весьма демократичную.

Выше мы писали о том, что соответствия между реле и стартером не всегда соблюдается. Учтите, что популярные производители стартеров могут использовать реле сторонних фирм. Наиболее известными производителями стартеров являются уже указанные выше фирмы, а также Magneton, Nikko, Mitsuba, Delco Remy, Cav, Poong Song, Delphi, Leece-Neville. По именам этих фирм также можно начинать поиск втягивающего реле в электронных каталогах известных интернет-магазинов.

Частые вопросы

Затронем несколько интересных вопросов, которыми могут задаться как водители-новички, так и весьма опытные автолюбители. Итак:

  • Какой ток втягивающего реле стартера? Ответ: многое зависит от модели стартера и реле, но в большинстве устройств через контакт управления проходит ток силой 15-20 Ампер. В некоторых случаях этот ток имеет большую силу, но также не забывайте, что сила тока зависит от того, чистые ли контакты;
  • Как разобрать втягивающее реле стартера? Ответ: данный вопрос затрагивался выше. При этом добраться можно и до «начинки» неразборного реле, хотя без специальных инструментов и нарушения целостности корпуса не обойтись.

Выше мы уже описывали процесс ремонта реле. Особых сложностей с этим не возникнет даже у неопытного автолюбителя. Проблема может возникнуть лишь в ходе поисков деталей реле. Впрочем, многие детали для ремонта можно найти у уже упомянутой датской фирмы Cargo. Большая часть предлагаемых фирмой товаров произведены в странах Юго-Восточной Азии, однако их качество довольно высоко.

Вывод

Мы рассмотрели основные неисправности втягивающих реле стартеров, а также задались вопросом выбора нового устройства. С учетом высокой живучести реле, обусловленной простотой и продуманностью конструкции, оно может служить ровно столько, сколько прослужит транспортное средство. Впрочем, поломка тоже не является редкостью. Так как речь идет об автомобильных электрических цепях, проблема может крыться не в самом реле, а других компонентах системы.

Учитывайте, что даже качественно отремонтированное реле редко служит свыше 2-3 лет. Так как оригинальное реле стоит немалых денег, в ремонте есть смысл – запчасти все равно стоят небольших денег, а с ремонт разборного реле легко осуществить. Если ремонт не дал никакого результата, стоит обратиться на СТО, вместо того чтобы покупать новое устройство и проверять работоспособность системы с его помощью.

Втягивающее реле стартера — как оно работает

Доброго времени суток, уважаемые читатели! В жизни каждого водителя бывали моменты, когда автомобиль отказывался работать и сдвинуть его с места можно было только с применением физической силы. Причин этого явления насчитывают множество, но что бы точно узнать в чем дело нужно провести диагностику транспортного средства. Если Ваш автомобиль не реагирует на запуск, в первую очередь следует проверить исправность электропроводки, аккумулятора и если с ними все хорошо, то вполне реально найти поломку в работе стартера.

Наиболее уязвимое место этой детали — втягивающее реле, о принципе работы которого знают немногие. Этот довольно маленький узел оказывает огромное влияние на начало работы мотора и в случае выхода из строя отдельных его составляющих, машина окажется полностью «парализована». Поэтому, в данной статье мы расскажем об особенностях работы втягивающего реле, возможных поломках и способах их устранения.

1. Функции втягивающего (тягового) реле стартера

Прежде чем начать обсуждение темы, стоит понимать о чем конкретно пойдет речь. Дело в том, что в конструкцию стартера входят два реле, отвечающих за его работу. Первое обеспечивает его включение и находится в моторном отсеке (в зависимости от модели автомобиля может либо монтироваться в общий блок реле, либо находиться в отдельном корпусе), а второе (тяговое реле) устанавливается на стартере и выполняет следующие функции:

— при запуске двигателя, синхронизирует работу узлов (цепей) стартера;

— способствует перераспределению электроэнергии между мотором стартера и электромагнитом реле;

— подводит шестерни бендикса к зубцам венца маховика, а после старта — возвращает ее на прежнее место. Эту функцию, многие автолюбители, считают основной в работе данного устройства.

Практически во всей автомобильной литературе, описанное устройство называется «тяговым реле стартера», однако, в народе его называют еще «втягивающим», имея ввиду туже самую деталь. Первым человеком, изобревшим втягивающий механизм, в далеком 1912 году, стал Чарльз Кеттеринг, один из основателей, популярной и в наши дни, фирмы Delco.

Именно благодаря ему, в то же время, с конвейера сошел первый автомобиль оборудованный таким устройством и, соответственно, электрической системой зажигания. Подобное нововведение, при запуске транспортного средства, позволило отказаться от использования специальной ручки (кривого стартера), которая помещался в шкив коленчатого вала и требовала немалых усилий для запуска двигателя.

Для того, что бы мотор начал работать (запустился), нужно обеспечить вращение коленвала ровно до того момента, пока в камерах сгорания не начнет воспламенятся горючая смесь. Если двигатель исправен, то на это действие затрачивается всего несколько секунд времени. Соответственно, в случае неисправной работы любой из частей стартера, отвечающего за вращение коленчатого вала, последний не будет двигаться, а в камерах сгорания не начнет гореть топливо и автомобиль никуда не поедет. Более конкретно, о принципе работы тягового реле стартера, а также возможных поломках этого устройства мы Вам дальше и расскажем.

2. Конструкция и система работы втягивающего реле

Данное устройство имеет относительно несложную конструкцию и включает в себя следующие детали: корпус, контакты реле, контактный диск, магнит с втягивающей и удерживающей обмоткой, сердечник (якорь), имеющий шток реле стартера и шток привода его вилки, возвратные пружины.

Основная часть этого реле — втягивающая катушка цилиндрической формы (образует электромагнит), внутри которой размещается подвижный якорь (сердечник), а сверху наматываются витки удерживающей катушки. С одной стороны сердечника размещен выходящий за габариты корпуса шток, который толкает вилку стартера и имеет на конце отверстие или перекладину (в зависимости от модели автомобиля). С другой стороны — находится шток, на конце которого находится контактный диск реле стартера. Корпус втягивающего реле представляет собой чашку с изоляционного материала, в которую вдавлены два контакта с нарезанной резьбой (к ней, гайками крепятся клеммы). Обычно, между контактами, на внешней части крышки, расположен бортик, который не допускает короткого замыкания. Сама крышка, с помощью винтов, так крепится к торцу реле, что его контакты выходят напротив контактного диска, размещенного на штоке сердечника.

Тяговое реле стартера прочно соединяется со стартером и располагается над ним. В случае необходимости, его довольно просто можно снять, но перед этим придется демонтировать сам стартер. В основном, все производители такой детали предлагают ее в двух вариантах: разборном (подлежит диагностике и ремонту) и не разборном, которое при поломке придется полностью заменить.

Весь процесс работы втягивающего реле стартера проходит следующим образом. Замыкание контактов в замке зажигания, приводит к срабатыванию реле стартера (обычное, размещено в монтажном блоке), которое, в свою очередь, от аккумулятора посылает напряжение на втягивающую обмотку. Таким образом, создается магнитное поле, под действием которого якорь попадает во внутрь обмотки, выполняя при этом сразу несколько действий: с одной стороны, с помощью штока заставляет вилку стартера начать движение и сдвинуть обгонную муфту (бендикс), способствуя зацеплению шестерни стартера с венцом маховика, а с другой, помогает закрепленному на штоке диску, замкнуть контакты втягивающего реле стартера.

Таким образом, в процессе движения якоря, стартер соединяется с маховиком и сразу же подключается к аккумулятору. После этого, по мотору стартера проходит ток и он начинает двигаться, а через несколько секунд запускается двигатель автомобиля.

Когда стартер включается, втягивающая катушка отключается и ток переходит на удерживающую катушку, одна из функций которой состоит в удержании крайнего положения якоря. Использование такой катушки способствует снижению мощности, потребляемой втягивающем реле, так как на сдерживание якоря затрачивается намного меньше энергии, чем нужно было бы для его втягивания. В результате, общие затраты заряда аккумуляторной батареи, при пуске двигателя, значительно снижаются.

После того, как двигатель машины запустился (для этого используется ключ зажигания), цепь стартера разрывается, обмотка тягового реле обесточивается и благодаря действию пружины якорь возвращается в начальное положение, а следом за ним бендикс и контактный диск отводятся от контактов реле стартера. Последний одновременно отсоединяется от маховика мотора и отключается от аккумулятора. Все последующее время эксплуатации транспортного средства, втягивающее реле стартера, да и он сам, никак не участвуют в работе силового агрегата автомобиля.

3. Диагностика и ремонт устройства

Втягивающее реле стартера имеет довольно простую конструкцию и, обычно, отличается долговечностью. Но не смотря на это, из-за частых больших нагрузок на данный узел и учитывая работу реле стартера с высоким напряжением (может достигать несколько сотен ампер) в работе втягивающего реле периодически возникают проблемы, обусловленные появлением характерных неисправностей. Чаще всего, они выражаются в следующем:

— силовые контакты реле стартера, со стороны контактного диска могут подгорать;

— втягивающая или удерживающая обмотка, время от времени, способна обрываться;

— деформирование возвратной пружины;

— короткое замыкание в обмотках;

— прочие механические повреждения в отдельных деталях втягивающего реле.

Однако, прежде чем сделать окончательный вывод о существовании поломки и планировать ремонт, следует провести полную диагностику реле. Тут важно помнить, что действует данная деталь как электромагнит: после того как на обмотки стартера подается напряжение, реле начинает притягивать к себе вилку стартера, которая подвигает бендикс, тем самым способствуя его сцеплению с маховиком. При выполнении таких действий происходит замыкание всех контактов, подающих напряжение на обмотки. В случае сбоя, хотя бы в одной части этого процесса, автомобиль не заведется. Что бы определить место неисправности, в первую очередь, следует руководствоваться внешними ее проявлениями.

Если стартер не крутит , то тут возможны два варианты: либо втягивающее реле срабатывает, но стартер не вращается, либо втягивающее реле и стартер вовсе не работают. Срабатывание реле определяют по характерному щелчку, который появляется в момент втягивания якоря вовнутрь. Если повернув ключ, Вы его услышали, то реле исправное, а если нет — значит оно не работает или же на него не поступает ток.

В случае, когда втягивающее реле работает, но стартер не крутит, причина может крыться в подгорании силовых контактов реле. Так это или нет, Вы можете проверить с помощью любого металлического предмета (например, отвертки), замкнув им выступающие части контактов. Если после этого, стартер начинает вращаться — значит проблема действительно подгоревших контактах, а если нет, то скорее всего причина в самом стартере.

Бывает, что одновременно и стартер, и реле не работают. Причины, в этом случае, могут быть как внешними, так и внутренними (например, обрыв цепи включения, неисправность замка зажигания, обрыв обмотки втягивающего реле, когда катушка теряет контакт с «массой» и т.д.).

Если втягивающее реле работает, но при этом слышен стук или дребезг, значит причина проблемы в плохом контакте обмотки(ок) с «массой». Проверить их не сложно, для этого всего лишь нужно измерить сопротивление с помощью омметра. Как правило, сопротивление втягивающей обмотки составляет примерно 0,55 Ом, в то время как удерживающей — 0,75 Ом. Сопротивление, меньше этих показателей, говорит о наличии короткого замыкания внутри обмотки, а слишком большое — о ее плохом контакте с массой или клеммами.

Если Вы заподозрили, в какой-то из обмоток обрыв, проверить эту догадку можно воспользовавшись пробником из батарейки или лампочки: если подключив лампочку к обмотке она горит — все нормально, а если нет, значит имеется обрыв. Недостатком этого способа является отсутствие возможности определения короткого замыкания, так как незначительная разница в сопротивлении, практически никак не влияет на яркость свечения лампочки.

Обнаружив поломку, втягивающее реле можно либо отремонтировать, либо заменить. Однако, здесь ключевую роль играет вид его конструкции: если в транспортном средстве установлено не разборное реле, то помочь Вам сможет лишь покупка и установка новой детали, а если его можно разобрать, то вполне реально устранить проблему самостоятельно. Для этого, прежде всего, нужно демонтировать реле со стартера и разобрать, в отдельных случаях придется отпаять выводы обмоток. Далее стоит проверить все его контакты, в случае обгорания — зачистить до блеска с помощью наждачной бумаги. Если Вы увидели отпавшие от корпуса выводы обмоток, стоит припаять их обратно. Кроме этого, можно заменить отдельные изношенные части втягивающего реле.

После проведения такого нехитрого ремонта, деталь сможет исправно прослужить еще несколько лет, правда, намного проще купить новое реле и не играться со старым, тем более, что стоит оно не так уж и дорого.

Подписывайтесь на наши ленты в таких социальных сетях как, Facebook, Вконтакте, Instagram, Pinterest, Yandex Zen, Twitter и Telegram: все самые интересные автомобильные события собранные в одном месте.

Втягивающее реле стартера — как проверить, заменить или отремонтировать

Для чего необходимо втягивающее реле стартера?

На всех автомобилях отечественного производителя втягивающее реле выполнено в одном блоке со стартером двигателя. Втягивающее реле имеет вид цилиндра и размеры, которые меньше стартера в два раза.

Втягивающее реле предназначено для управления муфтой свободного хода. Дело в том, что на конце муфты установлена шестерня, которая и воздействует на маховик двигателя. Эта шестерня нужна лишь для запуска двигателя и ее вращение, на протяжении всей работы двигателя недопустимо. Это может привести к различным поломкам стартера и даже бортовой сети автомобиля (так как стартер начнет работу в генераторном режиме). Чтобы этого не произошло, втягивающее реле выдвигает муфту лишь на тот момент, когда необходим запуск двигателя. После запуск двигателя и возврата ключа в исходное при включенном зажигании положении, втягивающее реле прекращает свою работу и муфта, под действием пружины возвращается в исходное положение, убирая шестерню от воздействия маховика двигателя.

Втягивающее реле имеют следующую особенность работы: ее электрическое соединение построено таким образом, что запуск электродвигателя стартера может быть произведен только после срабатывания этого реле. Это нужно, чтобы исключить перелом зубьев шестерни маховика или муфты при встрече с вращающейся шестерней муфты.

Принцип работы реле стартера

Чтобы понять, как производится ремонт втягивающего реле, необходимо знать его устройство и принцип действия. Реле, в свою очередь, представляет собой катушку с якорем.

Катушка реле встает под ток, возникший при замыкании цепи. Вокруг катушки возникает магнитное поле, которое воздействует на якорь реле. Якорь начинает двигаться в сторону катушки, сжимая возвратную пружину, и воздействует на рычаг, который выталкивает муфту свободного хода. После запуска двигателя, цепь катушки реле размыкается и магнитное поле, которое удерживало якорь, исчезло и якорь, под действием возвратной пружины, возвращается в исходное положение и устанавливает муфту свободного хода обратно.

Как проверить втягивающее реле

Часто случается такое, что двигатель автомобиля отказывается запускаться. Никакого вращения не происходит, а стартер отказывается реагировать на любой поворот ключа в замке зажигания. Чтобы убедиться, что втягивающее реле не работает, вначале необходимо провести проверку стартера целиком.

Вначале, попробуйте завести автомобиль обычным способом с помощью поворота ключа в замке зажигания. Если при этом слышны характерные щелчки, значит, втягивающее реле исправно и неисправность кроется только в самом стартере.

Если щелчки не происходят, возьмите любой провод и замкните два контакта на концах втягивающего реле (зажигание должно быть включено, а автомобиль должен стоять на нейтральной передаче). Если стартер начнет свое вращение, то он исправен, а проблема коснулась только втягивающего реле.

Если проверка реле непосредственно на автомобиле затруднена, то его можно проверить и при снятии стартера. Для этого его несильно зажимают в тисках. К контактному выводу реле присоединяют плюс аккумулятора, а к корпусу стартера минусовую клемму.

При замыкании контактов должен происходить резкий характерный щелчок, а шестерня должна выдвигаться наружу. Если этого не происходит, то реле неисправно. А в случаях, когда реле срабатывает достаточно медленно, неисправность кроется в подгорании его контактов.

Неисправности реле

После проведения проверки тягового реле, возможно обнаружение следующих неисправностей: залипание и подгорания контактов, различные перегорания катушки и прочие дефекты деталей из-за обычного износа деталей.

Признаки неисправного реле:

1. При повороте ключа слышен звук вращающегося стартера, но двигатель при этом не раскручивается.

2. Как только двигатель запустился, стартер продолжает свое вращение. Об этом свидетельствует характерный звук.

В основном, все реле, применяемые в стартерах, имеют схожую конструкцию. Их различие заключается только в размерах, способах крепления и конструктивных особенностях сердечника.

Видео — Ремонт (замена) втягивающего реле своими руками

В большинстве случаев, втягивающее реле является неремонтопригодным и подлежит замене. Дело в том, что этот узел неразборный и является расходным материалом. Однако, в некоторых случаях можно провести чистку подгоревших или загрязненных контактов. В случае, если прогорела сама катушка, то втягивающее реле подлежит только замене. Проводить ремонт катушки реле может только профессиональный электрик. Если сделать это самостоятельно, то есть риск вызвать короткое замыкание в электрической проводке автомобиля.

Порядок выполнения замены:

1. Перед разбором необходимо выполнить очистку узла от пыли и грязи. Это нужно для того, чтобы в внутрь не попали частицы, которые могут нарушить работу стартера.

2. Открутите гайку, которая расположенная на болте реле. После этого, снимите с болта специальный контакт.

3. Выкрутите винты, которые осуществляют крепеж тягового реле к массе, затем демонтируйте старый узел.

4. Установите новое втягивающее реле и проведите все операции в обратной последовательности. После монтажа реле проверяется перечисленными выше способами, и стартер монтируется на автомобиль.

На этом, проверка и ремонт втягивающего реле завершено. Стоит еще раз напомнить, в случае, если вы собираетесь провести именно ремонт втягивающего реле, то доверьте эту работу грамотному автомобильному электрику. В случае замены можно обойтись и собственными силами. 

Как проверить втягивающее реле стартера — Информация — autoshop98.ru

Несмотря на сравнительною простоту конструкции и принципа работы, проверку втягивающего реле стартера невозможно выполнить без определенных знаний и четкого понимания его устройства. Благо, этот узел не является самым часто ломающимся даже на отечественных автомобилях. Многие автолюбители годами даже не подозревают, что на стартере есть какое-то там втягивающее реле. Оно их просто никогда не беспокоило.

Тем не менее, втягивающее реле стартера – это нагруженный силовой узел. Через него проходят большие токи (пусковые), контакты обгорают, его обмотки подвергаются нагреву и замыкают, либо же уходят в обрыв. Не исключен и механический износ. Поэтому вопрос о том, как проверить втягивающее реле стартера, все же, возникает. А некоторым так «везет», что приходится «общаться» с этим узлом с завидной частотой.

Задача этой статьи – детально рассказать о том, как проверить втягивающее реле, зная его устройство и принцип работы. Дополнительно рассмотрены основные признаки неисправности этого узла, по возникновению которых, собственно, и должна происходить проверка. Процедура эта довольно простая, и справятся с ней даже начинающие любители починить машину своими руками. Главное – это внимательно следовать инструкции. Иначе фейерверка и травматизма не избежать.

Устройство и принцип работы втягивающего реле

Начать знакомство с втягивающим реле стартера, пожалуй, стоит с его общего вида на своем штатном месте. Воспользуемся для этого подходящим рисунком.

 

Как видим, втягивающее реле находится непосредственно на автомобильном стартере. Для него в корпусе предусмотрен отвод, в котором он и закрепляется двумя или тремя винтами. Механически втягивающее реле связано с частью стартера, которая называется обгонная муфта (в народе – бендикс). Сделано это для того, чтобы заводить шестерню перед бендиксом в зацепление с венцом маховика коленчатого вала двигателя.

Перемещение шестерни с бендиксом происходит благодаря тому, что якорь втягивающего реле в буквальном смысле слова втягивается, и через приводной рычаг воздействует на обгонную муфту. То есть, толкает ее с шестерней к маховику до их зацепления. Обратное действие, которое необходимо после успешного запуска двигателя, выполняется тоже втягивающим реле. Его якорь выдвигается, толкает рычаг, и оттягивает бендикс с шестерней в исходное положение.

Однако это не все, что делает втягивающее реле. Описанная функция – это как раз функция, из-за которой это устройство называют втягивающим. А вот реле его называют потому, что оно в нужный момент коммутирует «плюс» аккумуляторной батареи с самим стартером, благодаря чему и происходит вращение его якоря. Обратное действие – размыкание «плюса» АКБ и стартера – тоже выполняет реле втягивающего устройства.

Чтобы прояснить картинку, рассмотрим втягивающее реле изнутри.

 

За все механические и коммутационные функции втягивающего реле «отвечает» его шток, называемый также якорем. Втягивается он за счет воздействия электромагнитного поля, создаваемого обмоткой 2. Одновременно с втягиванием штока происходит коммутация силовых выводов 6 и 8 при помощи пластины 9.

Когда шток втянут, бендикс с шестерней вошли в зацепление с маховиком, а к стартеру через силовые выводы пошел пусковой ток, в таком положении втягивающее реле удерживается за счет воздействия электромагнитного поля, но уже создаваемого так называемой удерживающей обмоткой. На рисунке она под номером 3. Втягивающая же обмотка в это время уже отключена.

Когда двигатель запускается, водитель отпускает ключ зажигания, и удерживающая обмотка прекращает вырабатывать электромагнитное поле. Под воздействием пружин 5 и 7 шток с силовой пластиной возвращаются в исходное положение.

Рассмотрим вкратце, как работает электрическая часть втягивающего реле. Воспользуемся для этого вот такой замечательной и наглядной схемой.

 

Здесь буквами ЭД обозначен стартер со своими обмотками. Контакт реле – это та самая силовая пластина, которая толкается штоком во время срабатывания втягивающего. Провод, идущий от замка зажигания, подсоединен к втягивающему реле посредством так называемого управляющего контакта (тонкий плоский контакт). Маленькими кружками обозначены силовые контакты, которые мы можем видеть снаружи втягивающего реле.

Втягивающая и удерживающая обмотки на рисунке соединены точкой возле одного из силовых контактов. На реальном устройстве эту точку можно увидеть прямо на втягивающем реле в виде металлической перемычки. Это важно запомнить, так как путать силовые контакт втягивающего рее при проверке – нельзя. Вот как это выглядит в жизни. Здесь верхний силовой контакт соединен перемычкой с управляющим контактом. Нижний – соединяется со стартером толстым проводом, который можно видеть уходящим внутрь корпуса стартера.

 

В исходном положение ток от батареи не проходит ни к стартеру, ни к обмоткам втягивающего реле. Когда водитель поворачивает ключ зажигания, срабатывает втягивающая обмотка, так как она «посажена на землю» через обмотки стартера. Удерживающая обмотка «посажена на землю» на прямую, а потому срабатывает в этот же момент.

Далее под воздействием штока замыкается контакт реле. Поскольку из-за этого пришедший с замка зажигания «плюс» замыкается на такой же «плюс» через контакт реле – втягивающая обмотка, выполнив свою часть работы, отключается. Удерживающая же обмотка продолжает удерживать шток втягивающего реле. Поскольку контакт реле замкнут, на обмотки стартера с АКБ идет пусковой ток, и его якорь начинает вращаться. Двигатель запускается.

Когда водитель отпускает ключ замка зажигания, удерживающая обмотка обесточивается, контакт реле размыкается, и обесточивается и стартер. Все возвращается в исходное положение.

Признаки неисправности втягивающего реле

У втягивающего реле стартера может быть, как минимум, одно из семи неисправностей:

  1. Обрыв втягивающей обмотки. При такой поломке втягивание якоря не происходит. Соответственно, не вводится в зацепление приводная шестерня с маховиком, а также не замыкается силовой контакт реле. Признак такой неисправности – стартер при повороте ключа зажигания не подает никаких признаков жизни.
  2. Обрыв удерживающей обмотки. Эта поломка приводит к тому, что шестерня вводится в зацепление с маховиком, но сразу же после замыкания силового контакта реле – возвращается в исходное положение под воздействием пружины штока втягивающего устройства. Удерживать его во втянутом положении, так сказать, некому. Признак такой неисправности – стартер клацает, но не крутит.
  3. Межвитковое замыкание втягивающей обмотки. Случается довольно редко. А если и случается, то из-за сильных перегревов быстро переходит в поломку, которая называется – обрыв обмотки. Соответственно, выявить межвитковое замыкание, как правило, никто не успевает, пока обмотка не уйдет в разрыв и не проявится в виде симптома, описанного в пункте 1. В целом, может проявляться в том, что у втягивающего «не хватает» сил ввести шестерню в зацепление с маховиком.
  4. Межвитковое замыкание удерживающей обмотки. Тоже определяется редко, поскольку обычно втягивающее насилуется до тех пор, пока обмотка не уходит в обрыв из-за постоянных перегревов. А так – проявляется в том, что стартер по звуку клацает, но не крутит. Причем, клацает полноценно, что свидетельствует о полном заходе шестерни в зацепление с маховиком. Но из-за слабости удерживающей обмотки все тут же возвращается в исходное положение, и стартер в итоге «молчит».
  5. Оплавление силового контакта. При такой поломке стартер может вполне успешно запускать двигатель, но будет наблюдаться его тяжелая работа. То есть, из-за плохого контакта стартер будет получать мало пускового тока, и в итоге крутить двигатель вяло, туго и так далее. Заканчивается такая поломка, как правило, либо залипанием силового контакта, либо его разрушением.
  6. Залипание силового контакта. Эта неисправность довольно неприятная, и в случае возникновения может навести панику даже среди опытных автовладельцев. Когда силовой контакт залипает – стартер продолжает работать даже после того, как двигатель запущен (или нет) и ключ зажигания отпущен. Разомкнуться под воздействием пружины он не может, соответственно, стартер «молотит» до тех пор, пока либо не сгорит к чертям, либо пока водитель не сориентируется, и не скинет клемму с аккумулятора.
  7. Дефекты механического характера. Достаточно редко, но бывает, что шток втягивающего не может нормально двигаться из-за засорения. Также, если внутрь втягивающего попадает влага, зимой она замерзает, и в итоге втягивающая обмотка не может сдвинуть шток со с исходного места. Стартер – молчит.

Теперь, когда мы выяснили, как все работает и ломается, можно переходить к вопросу – как проверить втягивающее реле стартера.

Проверка втягивающего реле с помощью мультиметра

Перед тем, как проверить втягивающее реле, его необходимо вместе со стартером снять с машины. К сожалению, на вопрос – как проверить втягивающее реле стартера, не снимая – ответы дать сложно. Естественно, некоторые его неисправности можно выявить и на слух. Но, если таковые обнаружены – все равно придется все снимать с машины, и проверять уже на столе.

Исключением является разве что только замерзания втягивающего, в которое попала влага. В таких случаях достаточно просто отогреть этот узел при помощи фена или бытового обогревателя. Понять, что втягивающее замерзло, довольно просто. Если до этого все работало нормально, а с наступлением крепкого мороза стартер вдруг перестал подавать признаки жизни – логичнее всего будет предположить, что втягивающее замерзло, а не вышло из строя с другими описанными неисправностями. Хотя бывает всякое.

Также перед тем, как приступить к терзанию втягивающего реле, убедитесь, что:

  • АКБ заряжена и не является причиной неудовлетворительной работы стартера.
  • Между аккумуляторной батареей и стартером есть нормальный электрический контакт – силовые провода надежно закреплены и не окислились.

Демонтаж. Итак, сняли стартер с машины. Далее отсоединяется силовой провод, идущий от силовой клеммы втягивающего в стартер. Само втягивающее реле необходимо снять с корпуса стартера. Для этого, в зависимости от модели, нужно отвернуть два или три винта.

Если на вашем автомобиле втягивающее закреплено на двух винтах – обязательно пометьте маркером ориентацию его корпуса, а также его штока. В некоторых стартерах втягивающее собрать обратно неправильно невозможно, но все равно, лучше перестраховаться, и поставить маркером пару рисок для ориентирования.

Теперь для того, чтобы проверить втягивающее реле стартера на предмет обрыва обмоток или проблем с силовым контактом, понадобится только мультиметр с режимом прозвонки.

Проверка втягивающей обмотки

Осуществляется в следующей последовательности:

  1. Мультиметр включается в режим прямой прозвонки.
  2. Один из щупов прибора надежно подсоединяется к управляющему контакту втягивающего реле. Он плоский, и от него во время снятия стартера вы отсоединяли тонкий провод, идущий от замка зажигания.
  3. Второй щуп мультиметра подсоединяется к силовому контакту с перемычкой.

В таком положении ток, вырабатываемый мультиметром, должен пойти через втягивающую обмотку. Соответственно, если прибор пищит – обрыва нет. Если молчит – втягивающая обмотка в обрыве.

Проверка удерживающей обмотки

Осуществляется в следующей последовательности:

  1. Мультиметр включается в режим прямой прозвонки.
  2. Один из щупов прибора надежно подсоединяется к управляющему контакту втягивающего реле. Он плоский, и от него во время снятия стартера вы отсоединяли тонкий провод, идущий от замка зажигания.
  3. Второй щуп мультиметра подсоединяется к корпусу втягивающего устройства. При этом нужно убедиться, что у щупа есть надежный контакт с металлом корпуса (очистить от грязи и зачистить наждачной бумагой небольшой участок).

В таком положении ток, вырабатываемый мультиметром, должен пойти через удерживающую обмотку втягивающего реле стартера. Соответственно, если прибор подает звук – обмотка целая. Если мультиметр молчит – удерживающая обмотка втягивающего реле стартера в обрыве.

Проверка силового контакта реле

Цель этого этапа проверка втягивающего реле стартера заключается в том, чтобы убедиться, что при срабатывании втягивающей и удерживающей обмоток через силовой контакт идет пусковой ток на обмотки стартера.

Проверка выполняется в следующей последовательности:

  1. Мультиметр включается в режим прямой прозвонки.
  2. Первый щуп прибора подсоединяется к любому из силовых контактов втягивающего реле (их два, и они оба с резьбой).
  3. Второй щуп подсоединяется к оставшемуся силовому контакту.
  4. На шток (якорь) втягивающего реле необходимо воздействовать таким образом, чтобы он до упора вошел внутрь. Чтобы было удобнее это делать, щупы мультиметра следует закрепить на силовых контактах при помощи зажимов типа крокодил.

В таком положении, когда мы уже подключили прибор к силовым выводам, но не воздействовали на шток втягивающего устройства, пластина реле не должна быть прижата к силовым контактам, и ток через них течь не должен. Если же еще до нажатия на якорь втягивающего мультиметр сработал и показал замкнутую цепь – это указывает на то, что силовой контакт реле находится в залипшем состоянии. Об этом еще до разборки стартера должна была свидетельствовать непрерывающаяся работа стартера даже после отпускания ключа зажигания.

Если с силовым контактом реле все в порядке, то при описанном выше нажатии на шток втягивающего устройства мультиметр должен показать замкнутую цепь. Если в таком положении звука прозвонки нет – контактная пластина по каким-либо причинам не замыкает силовые контакты. До разборки и проверки эта поломка должна была проявить себя так – при повороте ключа зажигания мы слышим щелчок, но стартер не крутит двигатель.

Если во время описанной проверки мультиметр не запищал, но показал какое-то сопротивление на своем дисплее, то это означает, что реле проводит ток плохо. Происходит такой дефект, чаще всего, из-за оплавления контактных площадок, в результате чего значительно уменьшается сечение этого участка цепи, а сопротивление повышается. Стартер, при этом, мог и крутить двигатель, но еле-еле, вяло и неуверенно.

К сожалению, большинство втягивающих устройств невозможно разобрать для ремонта без варварских методов. Но, если вам это удастся, то вы увидите там примерно следующую картину.

 

Так выглядят оплавленные и сильно окислившиеся силовые контакты реле. Понятно, что пусковой ток, который, между прочим, более сотни ампер, нормально протекать через такое соединение не сможет. Если вообще сможет. В общем, если втягивающее устройство показывает при проверке дефект контакта реле, и его удалось аккуратно раскурочить – площадки можно очистить от оплавленного шлака и окислов, и, возможно, после удачной сборки все это заработает лучше, чем новое.

К сожалению, производители преобладающего большинства автомобильных стартеров считают, что обычному пользователю внутри втягивающего устройства смотреть не на что. Соответственно, делаются эти устройства абсолютно неразборными. С другой стороны, стоит втягивающее не так уж дорого. По крайней мере, куда дешевле, чем стартер в сборе. К тому же, ко многим иномарочным стартерам народные умельцы уже давно нашли альтернативную замену среди копеечных втягивающих устройств отечественного производства.

Поэтому, если проверка втягивающего реле стартера показала, что ему капец – его проще купить новое, чем как-то пытаться ремонтировать. Хотя гуру автомобильного ремонта, как говорится, море по колено – они даже разорванный напополам болт со слизанной резьбой не выбрасывают, а чинят.

Силовые испытания втягивающего реле

Ну и последнее, что касается того, как проверить втягивающее реле стартера – это так называемые силовые испытания. Цель мероприятия – определить, надежно ли отрабатывает удерживающая обмотка. То есть, не выходит ли бендикс с шестерней из зацепления с маховиком до того, как стартер прокрутит коленчатый вал двигателя. Такое, к сожалению, тоже бывает. А происходит из-за того, что в удерживающей обмотке случается межвитковое замыкание. Соответственно, электромагнитное поле, которое она вырабатывает – слабое, и удержать шток втягивающего не в силах, так сказать.

Вторая цель – определить, хватает ли силы втягивающей обмотки, чтобы затянуть якорь втягивающего устройства. То есть, может ли втягивающее реле завести шестерню в зацепление с маховиком.

Чтобы провести эти самые силовые испытания втягивающего реле стартера, потребуется три толстых провода и аккумуляторная батарея. Суть проверки заключается в том, чтобы сымитировать работу втягивающего реле, и убедиться при помощи ровных и сильных рук – прочно ли втянутый шток втягивающего удерживается соответствующей обмоткой.

Важно!!! Описанные далее действия требуют четкого понимания того, что делается. Что еще более важно – нельзя тормозить, заставляя втягивающее реле слишком долго пребывать в рабочем положении. Медлительные действия неизбежно приведут к перегреву удерживающей обмотки втягивающего, после чего проверка на предмет наличия в ней обрыва, описанная выше – даст «положительный» результат.

Проверка на силу втягивающей обмотки осуществляется в следующей последовательности:

  1. Якорь втягивающего реле необходимо закрепить на столе при помощи вкрученного надежно самореза и куска проволоки (показано на фотографии ниже).
  2. Силовой контакт без перемычки подсоединяется к «плюсу» АКБ.
  3. С помощью второго провода «плюс» АКБ подается на управляющий (плоский) контакт.
  4. Якорь заталкивается до контакта силового реле. Для этого он упирается в саморез, к которому он же и закреплен проволокой.
  5. Теперь самое важное! «Минус» от АКБ подается на силовой контакт с перемычкой.
  6. Сразу после срабатывания реле пробуем усилием руки оттянуть втягивающее, пытаясь вывести из него шток (для этого он прикрепляется к саморезу проволокой).
  7. После подачи «минуса» не мешкаем – в течение 3-4 секунд убеждаемся, что якорь втягивающего надежно удерживается внутри, и размыкаем цепь (снимаем «минус»).

 

Если в пункте 6 удалось усилием руки вытянуть шток, значит удерживающая обмотка работает слабо, либо не работает вообще.

Если бы удерживающая обмотка была в замыкании с корпусом, то во время проверки мы бы увидели сильное искрение на контактах (оно и так будет, но незначительное). Кроме того, шток втягивающего под воздействием пружины выталкивался бы из втягивающего самостоятельно.

Важно!!! Если с первой попытки вы замешкались, и не успели понять, что к чему – повторную проверку выполняйте только по истечении 5-10 минут. Это необходимо для того, чтобы дать возможность удерживающей обмотке остыть (на корпусе ее нагрев может и не ощущаться).

Краткие итоги

В качестве итога остается только сказать, что проверить втягивающее реле стартера не так уж и сложно. Как правило, практически все его неисправности определяются по первичным симптомам еще до снятия стартера. Проверка же в снятом виде нужна для того, чтобы исключить другие возможные неисправности, которые могут и не относиться к втягивающему реле. Кроме того, силовая проверка дает возможность убедиться, что втягивающее реле еще «не ослабло», и отрабатывает полноценно.

Схожий материал

Топ-10 отличных семейных автомобилей с полным приводом

Как подготовить автомобиль к продаже

Как правильно перевезти собаку в машине

Меняем тормозные колодки на колесах своими руками

Принцип работы датчика давления в шинах: обзор, особенности и устройство

Обман на АЗС: как нам не доливают бензин и как уберечься от жульничества?

Самый дешевый способ очистки системы охлаждения автомобиля

Как продлить жизнь автомобильного аккумулятора

Удаление ржавчины с кузова автомобиля в домашних условиях

5 причин смерти АКБ зимой / Причины неисправности АКБ

Правильная зарядка АКБ дешевым зарядным устройством

Система рулевого управления автомобиля, её диагностика и ремонт

Как избавиться от запотевания стекол в автомобиле. Причины и способы их устранения.

Об автомобильных тормозах: история появления, правила ухода

Восстановление автомобильного аккумулятора

10 возможных причин почему низкое напряжение бортовой сети

Галоген или светодиод — что лучше. Сравнение по 15 критериям

ГУР vs ЭУР: что лучше — гидроусилитель или электроусилитель руля

Как проверить генератор автомобиля: 10 основных неисправностей

10 причин почему стартер еле крутит и пояснения к ним

Какой домкрат купить и как правильно его выбрать

Какой набор инструментов купить для автомобиля: 5 вариантов

История шин Bridgestone / Бриджстоун

История шин Матадор / Matador

Устройство втягивающего реле стартера и его замена + Видео » АвтоНоватор

Устройство втягивающего реле стартера – электрического магнита, который предназначен для подведения к венцу маховика шестерню бендикса, относительно простое. Поэтому ремонт данного механизма, часто называемого тяговым, любой более-менее опытный автолюбитель может осуществить самостоятельно.

Тяговое реле стартера – какие функции оно выполняет?

Втягивающий механизм был изобретен Чарльзом Кеттерингом. Именно благодаря ему уже в 1912 году было выпущено первое авто с таким устройством и системой электрического зажигания, что позволило отказаться от запуска мотора при помощи специальной ручки, помещаемой в шкив коленчатого вала. Описываемое нами приспособление предназначено для выполнения ряда важных функций, в частности таких:

  • синхронизация при запуске двигателя автомобиля функционирования цепей стартера;
  • перераспределение электрической энергии между мотором стартера и электромагнитным реле;
  • подведение шестерни бендикса к маховику, а после пуска – возвращение ее в начальное положение.

Интересующий нас механизм не стоит путать с реле включения стартера, которое располагается в моторном отделении.

Реле включения также имеет большое значение для функционирования стартера автомобиля, но оно выполняет совершенно иные задачи.

Принцип работы и устройство втягивающего реле стартера

Данный механизм состоит из следующих элементов:

  • электромагнит с удерживающей и втягивающей обмоткой;
  • корпус;
  • контакты;
  • якорь;
  • возвратная пружина (она находится в сердечнике устройства).

При подаче напряжения на обмотки втягивающего механизма (оно возникает при повороте ключа зажигания) создается магнитное поле, которое передвигает якорь стартера. Этот якорь в свою очередь давит на возвратную пружину и сжимает ее. Конец вилки, размещенный на противоположном от корпуса устройства конце, начинает перемещаться к маховику, что приводит к зацеплению шестерни с его венцом.

При этом происходит замыкание так называемых «пятаков» – пары контактов, встроенных в конструкцию рассматриваемого нами реле. В результате всех указанных движений удерживающая обмотка включается и передает на обмотку двигателя требуемый ток, который приводит во вращение зацепленный с шестерней маховик и вал.

Когда ключ зажигания возвращается в первоначальное положение (пуск двигателя произведен), «пятаки» размыкаются, механизм перестает получать электрическую энергию. Как результат – якорь, управляемый возвратной пружиной, становится в свое начальное положение, а маховик выходит из зацепления с бендиксом. Как видим, устройство тягового реле стартера простое, по этой причине принцип его работы может уяснить любой водитель.

Втягивающее реле стартера – ремонт устройства

Чаще всего ремонт реле требуется из-за:

  • разрушения материалов в результате их усталости и потери прочности;
  • сгорания обмотки стартера либо реле;
  • выгорания внутри механизма «пятаков».

При каждом цикле работы рассматриваемое нами устройство воспринимает критические нагрузки, что и становится причиной относительно быстрого выхода из строя его составных частей. Причем поломка вилки, контактов, обмоток приводит к тому, что мотор завести не удастся никаким способом.

Мнение эксперта

Руслан Константинов

Эксперт по автомобильной тематике. Окончил ИжГТУ имени М.Т. Калашникова по специальности «Эксплуатация транспортно-технологических машин и комплексов». Опыт профессионального ремонта автомобилей более 10 лет.

Чтобы не снимать и не разбирать тяговое реле стартера безосновательно, рекомендуется сначала провести его проверку на работоспособность. Тем более сделать это сможет даже новичок. Если стартер молчит и не подает признаков жизни, первое, что нужного проверить это втягивающее, и начать стоит с проводки путём прозвона, с помощью мультиметра нужно убедиться в целостности.
Если проводка исправна, далее следует убедиться, что реле работает. Для этого может потребоваться помощник, который повернет ключ зажигания, в этот момент должен послышаться щелчок, сигнализирующий о работе механизма. Если щелчка нет, реле следует заменить. Бывает и такое, что щелчок есть, но стартер молчит, в таком случае срабатывание реле не происходит преимущественно из-за подгоревших контактов (медные пятаки). Чтобы это проверить, нужно скинуть клемму, которая идёт от аккумулятора, вооружиться отвёрткой и замкнуть верхний и нижний контакт на реле. Если стартер заработал, значит, проблема во втягивающем реле.
Также можно проверить напряжение, поступающее на стартер при помощи мультиметра, однако этот метод позволяет определить источник проблемы (стартер или проводка с аккумулятором). Красный щуп подсоединяется к плюсовой клемме АКБ, чёрный к массе. После этого помощник должен повернуть ключ в замке зажигания в положение запуска, мультиметр в этот момент должен показать напряжение не ниже 12 В. Если показатели оказались меньше, значит, АКБ разряжена и попросту нет достаточной энергии для запуска мотора, однако напряжения достаточно для срабатывания механизма реле.

Вопрос того, как разобрать втягивающее реле стартера, не является проблемой, если речь идет о разборном устройстве. Оно соединено со стартером и располагается непосредственно над ним. Чтобы провести диагностику и ремонт механизма, следует обязательно демонтировать стартер, а потом произвести разборку реле.

Если же на вашем транспортном средстве установлено неразборное втягивающее устройство, «разложить» его полочкам не получится. Придется купить новое реле и установить его на место старого. Вся операция, учитывая простое устройство втягивающего реле стартера, займет у вас от силы 15 минут.

Оцените статью: Поделитесь с друзьями!

Втягивающее реле стартера: устройство, неисправности, ремонт

Сломался авто?
Мы все починим!
+7 (961) 014-5673
+7 (915) 732-0659
Звоните!

Втягивающее (правильно: тяговое) реле в автомобиле принимает ток от аккумулятора и передаёт его на стартер, выталкивая одновременно бендикс для сцепления с маховиком. В основе принципа его работы лежит обыкновенный электромагнетизм. В этой статье вы узнаете как работает втягивающее реле, почему оно ломается и что делать, если это произошло.

Устройство втягивающего реле

Составные части тягового реле:

  • корпус;
  • магнит с обмотками;
  • якорь;
  • контакты;
  • возвращающая пружина.

Электромагнит реле имеет две катушки возбуждения: одна вытягивает бендикс, а другая удерживает его до полной раскрутки коленчатого вала, после чего соединение бендикса с маховиком мотора размыкается.

Неисправности тягового реле

Причины, приводящие к поломке втягивающего реле просты: постепенный физический износ, иногда приводящий к частичному разрушению этого узла или выгоранию обмотки. Как правило, неисправности тягового реле легко определяются по следующим признакам:

  • После запуска двигателя стартер не прекращает вращаться;
  • Стартер не может запустить мотор;
  • Втягивающее реле стартера щелкает, но стартер не крутит.

Проверка втягивающего реле стартера

Наиболее точно говорит о том, что реле неисправно отсутствие сцепления бендикса стартера с маховиком двигателя. Признаками выхода из строя втягивающего реле является невозможность отключения стартера после запуска двигателя, при этом слышно характерное жужжание, то есть стартер вращается в холостую.

Для проверки стартера не обязательно снимать его. Достаточно просто любым металлическим предметом замкнуть контактные болты, расположенные в задней части тягового реле. Если стартер будет крутиться, значит действительно неисправно реле.

Ремонт втягивающего реле стартера

Вернуть неисправное реле в прежнее состояние не всегда оказывается возможным, поэтому скорее всего его придётся заменить. Но и в этом случае могут быть проблемы, так как многие подобные устройства делаются неразборными. К тому же трудно найти какие-либо детали реле в продаже. Единственное, что можно сделать — очистить контакты прерывателя от загрязнений. Если это не поможет, придётся пойти на замену.

Если при включении стартера щелкает втягивающее и гаснут приборы необходимо в первую очередь проверить состояние всех соединений и проводов системы запуска двигателя, так как причина неисправности может быть как в самом реле, так и в питающем его кабеле. Установить причину можно подав ток на управляющую клемму стартера. Если после этого отчётливо слышно, как работает стартер, надо проверять реле, проводку и замок зажигания.

В случае когда стартер не крутит, а втягивающее щелкает, это значит, что оно исправно и на него поступает ток. Искать причину того, что не заводится мотор нужно в другом. Возможно следующее:

  • “посажен” аккумулятор;
  • плохие контакты, повреждена проводка;
  • замок зажигания сломался;
  • нет контакта на щётках стартера.

То что вы слышите щелчки может означать, что втягивающее в порядке, просто АКБ находится на пределе своего заряда и чтобы крутануть стартер ему не хватает энергии. Ну и как видите, поводов к тому, что не всегда срабатывает втягивающее реле стартера может быть ещё много. Так что втягивающее может оказаться ни при чём.

Контакт, чистота и целостность изоляции, — вот три оплота исправности автомобильного электрооборудования. Вполне возможно, что неприятности со стартером возникли из-за неполадок щёточного узла, через который обычно подключен один из контуров возбуждения втягивающего реле. Если щётки нормальные, проверяйте всю систему зажигания по схеме, где-то недосмотрели. Если узел нуждается в замене, придётся купить втягивающее на стартер.



Втягивающее реле стартера

Втягивающее реле стартера – электромагнит, подводящий шестерню бендикса к зубчатому венцу маховика

Двигатель

Втягивающее реле стартера выполняет две основные функции. В момент запуска двигателя подводит соединенную с валом стартера шестерню к зубчатому венцу маховика, а когда мотор заведется – возвращает в исходное положение.

Бендикс. Втягивающее реле стартера.

История создания втягивающего реле стартера

Втягивающее реле – ровесник стартера. Идея использования электромотора, впервые появилась у американского инженера Чарльза Франклина Кеттеринга, одного из основателей продолжающей процветать и в наши дни фирмы Delco. Первый автомобиль с электрическим зажиганием, стартером и втягивающим реле появился в 1912 году. Изобретение системы зажигания Delco открыло женщинам дорогу в автомобилисты. Заводить двигатель с помощью «кривого стартера», то есть ручкой, вставляемой в шкив коленвала, им было не под силу.

Устройство и принцип работы втягивающего реле стартера

Втягивающее реле стартера устроено по принципу стандартного электромагнита. Состоит из двух обмоток (втягивающей и удерживающей), схемы включения стартера и сердечника с возвратной пружиной. Внешний конец якоря сопряжен с вилкой, работающей по принципу рычага. Зафиксированная на оси в центральной части вилка противоположным от втягивающего реле концом перемещает бендикс при срабатывании электромагнита, втягивающего сердечник.

В доме изобретателя электрического стартера и втягивающего реле Чарльза Кеттеринга был установлен первый в США домашний кондиционер

При повороте ключа зажигания (или нажатия соответствующей кнопки) на обмотки втягивающего реле подается напряжение. Возникает магнитное поле, перемещающее якорь. Якорь сжимает возвратную пружину. Противоположный от корпуса втягивающего реле конец вилки двигается по направлению к маховику, выталкивая шестерню, соединенную с бендиксом, до зацепления с зубчатым венцом маховика. В момент зацепления замыкаются контакты также интегрированной в конструкцию втягивающего реле схемы включения стартера. При этом более мощная, предназначенная для преодоления сопротивления возвратной пружины, втягивающая обмотка отключается, а якорь фиксируется в заданном положении с помощью удерживающей обмотки. После возвращения ключа зажигания (а автомашинах с «кнопочным» пуском – автоматически после старта двигателя) контакты, подающие напряжение на втягивающее реле, размыкаются, ток перестает поступать на обмотки и пружина возвращает якорь стартера в исходное положение, благодаря чему соединенный с ним вилкой бендикс выходит из зацепления с маховиком.

Особенности эксплуатации втягивающего реле стартера

Из-за того, что при каждом цикле работы втягивающее реле испытывает максимальную нагрузку большинство повреждений этого узла связано с выгоранием фрагментов электромагнитной схемы – обмоток и контактов – а также усталостью металла, приводящей к поломке вилки. При этом неисправности втягивающего реле стартера относятся к числу критических – завести двигатель в случае поломки невозможно.

Чтобы приобрести втягивающее реле для Ferrari самостоятельно, лучше всего отправиться в Дубаи — мировой центр поставок автомобильных запчастей

Впрочем, прежде чем выносить приговор самому реле, необходимо сначала проверить состояние цепи подачи напряжения на стартер. Характерные щелчки (якорь не попадает под действие удерживающей обмотки и возвращается пружиной в исходное положение до завершения цикла пуска) или «моргание» контрольных ламп на панели означают либо малый заряд аккумулятора либо плохой контакт на каком-то из участков цепи.

Planet Analog — Реле и соленоиды: электромеханические устройства

По мере развития автоматизации механические функции все больше и больше полагаются на электронное управление. Соленоиды обеспечивают электромеханический интерфейс для многих из этих приложений. Кроме того, многие электронные нагрузки часто включаются и выключаются. Реле — это форма соленоида, который переключает электронные нагрузки. Этот блог представляет собой введение в реле и соленоиды для новичков с несколькими историями о моем опыте работы с каждым из них.Детали реле и соленоидов подробно описаны в справочных материалах. Есть также ряд обучающих видео, которые можно найти, выполнив поиск «реле» и «соленоиды» на YouTube.

Соленоид — это в основном электромагнит, который создается путем подачи тока на катушку с проволокой. В физике мы изучаем «правило правой руки», которое определяет направление магнитного поля, когда ток течет в катушке с проволокой. Если согнуть пальцы в виде обернутой катушки, магнитное поле будет вытекать из кончика вытянутого большого пальца, когда ток течет из завернутых кончиков пальцев.

Когда якорь вставляется внутрь катушки, магнитная сила выталкивает якорь в направлении большого пальца, когда катушка находится под напряжением. Величина силы зависит от количества витков обмотки катушки, а также от величины тока, протекающего через катушку. Эта сила используется для всех видов нагрузок, включая включение пусковых двигателей двигателя, дверных замков с электроприводом и движущихся клапанов, таких как те, которые встречаются в автомате с газировкой. Этот «щелчок», который вы слышите, на самом деле является ударом якоря по клапану при приложении магнитной силы.

На самом деле в действии соленоида действуют две силы. Первая сила — это сила, необходимая для перемещения якоря. Вторая сила — это сила, удерживающая якорь на месте. Поскольку у большинства соленоидов есть пружина, удерживающая якорь, электрическая сила должна быть достаточно мощной, чтобы преодолеть силу пружины и сдвинуть или удерживать якорь на месте. Из этих сил удерживающая сила меньше начальной силы, прикладываемой для запуска якоря в движение.

Недавно я столкнулся с остановкой дизельного двигателя из-за плохого удерживающего тока якоря.Соленоид, который используется для управления топливным клапаном, выходит из строя из-за того, что топливный клапан не удерживается открытым. Соленоид также запрещает запуск из-за недостаточного открытия клапана. Последний отказ фактически связан с отказом как соленоида, так и реле. Взгляд на жгут проводов объясняет, как произошла неисправность. При трогании с места запускается соленоид, открывающий топливный клапан. Это требует большей мощности, чем ток, используемый для удержания клапана в открытом состоянии, чтобы двигатель работал. Дополнительная пусковая мощность включается реле.Комбинация контактов реле, образующих дугу и становящихся более резистивными, вместе с разъемами, также снижает напряжение, и система заземления снизила напряжение на соленоиде до точки, при которой он не вытягивает клапан достаточно далеко, чтобы запустить двигатель. . Результат — нехватка топлива при запуске. На верхнем фото (внизу) разъема с синей маркировкой показаны три провода на стороне соленоида разъема:

  1. Красный провод
  2. Белый провод включения якоря
  3. А черный провод массы

На верхней фотографии (вверху) разъема с синей маркировкой показаны три провода на стороне соленоида разъема

.

Однако обратите внимание, что зеленый удерживающий провод на стороне жгута проводов на нижнем фото намного меньше.Удерживающий ток должен быть меньше примерно на 10,7 вольт по сравнению с пусковым током при 12 вольт. Однако на каждом пути тока наблюдается слишком большое падение напряжения, что вызывает проблемы при запуске и работе. Замена соленоида и реле может решить проблему. В противном случае решением может быть снижение потерь напряжения в жгуте. Наконец, провод заземления проходит через жгут, что также добавляет дополнительные капли. Ответом может быть более прямой путь.

Реле — это соленоиды, которые перемещают набор контактов для включения цепи.Реле широко используются в автомобилях. По мере увеличения нагрузки текущие пути остаются локальными, а не идут до переключателей и управляющих сигналов под приборной панелью. Таким образом, реле используют сигналы низкого уровня для питания более высоких нагрузок.

Реле

часто являются более дешевым выбором, чем полупроводниковый переключатель и связанные с ним вспомогательные компоненты. Реле также изолированы, что означает, что схема переключателя может ссылаться на отдельное заземление, или переключаемые контакты могут выполнять переключение напряжения на стороне высокого напряжения.Для автомобильных применений реле предлагают ножевые клеммы с закрытым корпусом, который часто имеет монтажный язычок. Это упрощает подключение по принципу «включай и работай» по сравнению с полупроводниковой схемой, которая требует изоляции, способа защиты и монтажа печатной платы, а также подключения к корпусам полупроводников или дорожкам печатной платы.

Внутренний вид реле, включая контакт и катушку (из учебного пособия на Explainthatstuff.com)

Помимо изоляции по напряжению, реле позволяют переключать большие токи, в том числе для двигателей и других нагрузок.В результате может возникнуть дуга в переключателях, особенно для емкостных нагрузок, которые имеют большие начальные всплески тока. Неисправность контактов — частая проблема реле. В крайних случаях контакты фактически свариваются друг с другом. Это произошло однажды в комплекте внедорожных фонарей, которые я установил на свой грузовик. Мне пришлось съехать с автострады и выключить свет, чтобы не ослеплять встречный транспорт и не прожигать дыры в головах водителей, идущих впереди меня.

Другой вид реле — герконовое реле.Герконовые реле имеют осевую конструкцию, в которой контакт также обеспечивает натяжение пружины, что создает разрыв цепи. Катушка находится под напряжением, создавая магнитное поле, замыкающее контакты. Герконовые реле пропускают меньший ток, чем стандартное электромеханическое реле, но переключаются быстрее.

Схема герконового реле (изображение любезно предоставлено официальным документом National Instruments 3 )

У реле

есть срок службы, который зависит от количества циклов, которые испытывает реле.Чем больше размыкаются и замыкаются контакты, тем больше возникает дуга. Это снижает срок службы реле. В следующей таблице показано сравнение скоростей переключения реле, ожидаемого срока службы и допустимой нагрузки по току.

Сравнение типов и характеристик реле, из «Как выбрать правильное реле 3 », National Instruments

Реле переключения когда-то играли важную роль в регуляторах напряжения для автомобилей. Контакты реле включались и выключались в импульсном режиме для усреднения напряжения, поступающего от генератора.По этой причине многие владельцы классических автомобилей переходят свои генераторы на более надежные твердотельные версии. Релейные регуляторы напряжения были не только ненадежными, но и требовали трудоемкой процедуры настройки. Кроме того, многие из корпусов регуляторов напряжения были из проводящего металла, которые приходилось изолировать от мест их крепления с помощью резиновых втулок. Когда втулки высыхают, трескаются и состариваются, в регуляторе возникает короткое замыкание, вызывающее головную боль.

Реле

, опция для переключения нагрузки, дожили до эпохи интегрированных полупроводников.Они остаются жизнеспособной альтернативой для многих приложений. Соленоиды появляются во все большем количестве приложений, поскольку мир становится управляемым электронным способом. Надеюсь, вам понравилось это знакомство с реле и соленоидами, электромеханическими технологиями.

Артикул:

  1. Реле, документ Центра кранов ВМС США
  2. «Магнитная сила и поле — Вопросы», веб-страница
  3. «Как выбрать правильное реле», официальный документ National Instruments, дата публикации: 18 августа 2017 г.
  4. «Реле» Криса Вудфорда.Последнее изменение: 23 апреля 2017 г.

Что такое соленоид? — Блог Del City

Обзор

Часто при работе с приложениями с высокой силой тока соленоид включается в систему проводки. Это связано с их способностью управлять большой выходной мощностью при очень малой потребляемой мощности. Но что такое соленоид? Некоторые назвали бы их «релейными переключателями», но на самом деле это просто гигантские реле, а не переключатели, потому что они не могут быть задействованы вручную, как, например, тумблеры.Благодаря способности соленоида выдерживать такую ​​большую нагрузку, их можно рассмотреть, когда реле не может выдержать силовую нагрузку, необходимую для приложения (например, снегоочистители).

Дизайн

Соленоид будет иметь разное количество контактов в зависимости от его срабатывания и обязанности. В этом примере мы взяли изолированный сплошной соленоид с срабатыванием SPST (наиболее распространенный тип срабатывания):

В этом случае два меньших контакта в центре вверху являются клеммами управления катушкой.По сути, эти два меньших контакта для питания и заземления работают так же, как контакты 86 и 85 на реле.

Затем два больших контакта (по обе стороны от меньших) — это то, что управляет вспомогательной схемой. Эти более крупные контакты действуют как контакты 30 и 87 на реле. Поскольку это соленоид SPST, в этой цепи может быть активирован только один аксессуар.

Внизу вы увидите два монтажных кронштейна, которые, как вы уже догадались, помогают при установке.Но эти кронштейны также поддерживают заземление системы при использовании неизолированного соленоида.

Магнитное поле и функциональные возможности

Магнитное поле соленоида создается при подаче напряжения на катушку внутри. По сути, если вы откроете его, вы увидите катушку, обернутую вокруг сердечника соленоида и Т-образного вала или «плунжера».

Когда цепь активирована, катушка получает питание от силового контакта.Эта энергия перемещает головку плунжера, чтобы соединить два более крупных контакта (питание и заземление).

Если вы заглянете внутрь соленоида, то, когда он находится под напряжением, вы увидите, что головка плунжера втягивается, чтобы соединить два больших разъема, как показано выше.

После того, как это соединение установлено, питание передается на аксессуар. В большинстве случаев это приводит к состоянию активации.

Что касается соленоидов, то есть множество вариантов.Давайте посмотрим, какие ключевые моменты следует учитывать при выборе соленоида для вашей следующей сборки.

Заземленный и изолированный

Выберете ли вы заземленный или изолированный, во многом будет зависеть от того, над чем вы работаете. Если вы создаете собственное приложение, выбор за вами. С заземленным соленоидом у вас будет на один провод заземления меньше, что может упростить задачу. Однако, если вы заменяете соленоид в существующем приложении, чаще всего вам нужно заменить заземленный на заземленный и изолированный на изолированный.Исключением из этого правила является проблема с заземлением цепи. Допустим, изначально использовался заземленный соленоид, но из-за того, что монтажные кронштейны не находятся заподлицо с монтажным основанием или имеется значительная коррозия, цепь не заземлена. В этом случае лучшим вариантом может быть переход на изолированный, чтобы вы могли добавить новый провод заземления.

С первого взгляда относительно легко определить, изолирован ли соленоид или заземлен. Когда он заземлен, у него не будет заземляющего контакта, выходящего за пределы корпуса (скорее, он находится внутри), поэтому открыты только три клеммы (две большие и одна маленькая).Если он изолирован, вы увидите, что заземляющий контакт выступает с внешней стороны корпуса соленоида, поэтому вы увидите четыре клеммы (две большие и две маленькие), потому что вам нужно подключить заземляющий провод.

Непрерывный или прерывистый

При выборе между непрерывным или прерывистым режимом это просто зависит от того, как long будет работать ваше приложение. Например, чтобы завести автомобиль, требуется лишь небольшой промежуток времени, когда требуется скачок мощности: когда ключ вначале поворачивает зажигание, чтобы запустить двигатель.Поскольку требуется лишь короткий период времени, когда требуется сильный прилив энергии, лучше всего будет периодический. Однако, если у вас есть приложение, которое требует значительных затрат в течение длительного периода времени, лучше всего выбрать непрерывный. Это может быть, например, такое приложение, как охлаждающий вентилятор.

Вот эмпирическое правило при выборе между прерывистым и непрерывным режимом:

Прерывистый 0-15 секунд мощности при максимальном номинальном токе
Непрерывный 0-15 секунд питания при значительном значении силы тока, затем 15+ секунд при стабильном номинальном токе

Пиковая сила тока — это, по сути, максимальная сила тока, протекающая через цепь, и обычно используется только для первоначального запуска цепи.Другой стандартный номинальный ток в амперах, который вы часто будете видеть, — это постоянный номинальный ток.

При непрерывном режиме работы соленоид может оставаться включенным в течение длительного времени, но может выдерживать более низкие токи из-за продолжительности времени, в течение которого они остаются включенными. И наоборот, периодические режимы работы позволяют использовать более высокие токовые нагрузки в течение гораздо более коротких периодов времени.

Итак, вот что такое соленоид, как он работает и как выбрать его для следующей сборки. В следующий раз, когда вы начнете работу, подумайте о соленоиде для тех больших силовых нагрузок, когда реле просто не может выполнять эту работу.А если у вас есть вопросы, позвоните в нашу службу технической поддержки по телефону 1.800.654.4757.

Соленоиды непрерывного действия

и соленоиды обычного стартера: почему они не взаимозаменяемы

Соленоид стартера является важной частью систем вашего автопарка, так как без него вы не сможете запустить свои грузовики. Для запуска транспортного средства требуется большая мощность, и для управления этой мощностью необходим соленоид. Однако установка неправильного соленоида может иметь катастрофические последствия.

Вот базовое объяснение того, как работает стартер, различные типы соленоидов и почему они не взаимозаменяемы.

Как работает соленоид стартера

Представьте, что вы заводите их по утрам.

Столетие назад, чтобы запустить двигатель автомобиля, нужно было провернуть его вручную. Конечно, для удобства от этой системы быстро отказались. Однако, чтобы быть эффективным, пусковой механизм должен приводить двигатель в движение достаточно быстро, чтобы он самостоятельно генерировал мощность — сложная задача, поскольку для этого требуется много энергии.

Без современной системы зажигания и соленоида стартера толстые кабели должны были бы передавать питание от аккумулятора к зажиганию и снова от зажигания к стартеру. Система зажигания и соленоид стартера позволяют проложить только один кабель от батареи к соленоиду, при этом соленоид эффективно действует как плотина, не пропуская мощность до тех пор, пока зажигание не будет включено.

Когда зажигание переводится в положение запуска, соленоид стартера замыкает цепь, передавая питание на стартер, который затем вращает двигатель.Как только двигатель заработает своим ходом, водитель отпускает ключ, возвращая зажигание в положение «включено». Затем соленоид размыкает цепь и отключает питание стартера.

ЗАПРОСИТЬ ИНФОРМАЦИЮ

Обычная и непрерывная работа: в чем разница?

Соленоид непрерывного режима работает так же, как и обычный соленоид стартера, размыкая и замыкая цепь, чтобы выключить и включить поток мощности. Однако, как вы могли догадаться, в случае соленоида постоянного режима поток мощности более постоянен, тогда как соленоид стартера работает с перебоями.Мощность, передаваемая через соленоид непрерывного режима, также намного ниже, потому что он должен работать в течение длительных периодов времени. Соленоиды длительного режима обычно не нагревают никакую систему так сильно, как пусковая система вашего автомобиля, поэтому они также созданы, чтобы выдерживать гораздо более низкие уровни, чем соленоиды стартера.

Почему они не взаимозаменяемы

Поскольку они должны соответствовать требованиям самых разных типов систем, соленоиды стартера (прерывистые) и соленоиды непрерывного режима сильно отличаются, и их замена будет катастрофической.Соленоид непрерывного режима не рассчитан на то, чтобы выдерживать количество энергии, требуемой в системе запуска транспортного средства, а соленоид стартера не может выдерживать высокую температуру при непрерывной работе.

Использование неподходящего соленоида может привести к его очень быстрому сгоранию, что приведет к дополнительному ремонту. Чтобы свести к минимуму ремонт, сэкономить деньги и гарантировать, что автомобили вашего автопарка прослужат как можно дольше, важно всегда убедиться, что вы используете правильный соленоид для конкретного применения.

Свяжитесь с нами сегодня, чтобы обсудить ваши варианты или разместить заказ онлайн.

Признаки неисправного реле стартера и способы устранения? — Приемник-механик

Знание симптомов неисправного реле стартера спасет вас от застревания в глуши. Реле стартера — один из важнейших компонентов системы зажигания, о котором чаще всего забывают.

Этот критически важный компонент системы зажигания предназначен для передачи тока от автомобильного аккумулятора к соленоиду стартера, заставляя стартер запускать двигатель. Это означает, что он служит переключателем между стартером и соленоидом стартера в автомобиле.

Проблемы с реле стартера встречаются нечасто. Он редко выходит из строя, но когда это происходит, вы можете оказаться в затруднительном положении, потому что ваша машина не заводится. Однако несколько факторов могут вызвать отказ реле стартера. Они включают в себя плохие цепи, корродированные цепи, перемычки, влажные реле или даже устаревшие реле. Когда ваше реле стартера выходит из строя или начинает выходить из строя, вы заметите некоторые предупреждающие знаки, которые побудят вас вовремя исправить это.

В этой статье мы подробно рассмотрим эти признаки и рассмотрим, как определить, неисправно ли реле стартера.Мы также обсудим функции, причины их сбоев и способы их исправления или замены.

Что делает реле стартера?

Реле стартера — это компонент системы зажигания, специально разработанный для передачи энергии от автомобильного аккумулятора на соленоид стартера. Это означает, что это переключатель между соленоидом стартера и стартером.

Он предназначен исключительно для правильной передачи тока от автомобильного аккумулятора к стартеру. Большинство автовладельцев и механиков путают реле стартера с соленоидом стартера.Это два разных компонента системы зажигания.

Соленоид стартера работает как активная катушка соединителя, по которому электрический ток проходит от соленоида стартера к стартеру. Соленоиды стартера предназначены для зацепления шестерни стартера с зубчатым венцом двигателя.

Итак, как реле стартера, соленоид стартера и стартер работают вместе? Когда вы вставляете ключ в ключ зажигания, чтобы включить его, он активирует реле стартера, которое передает мощность на соленоид стартера, который передает мощность на двигатель стартера.

Реле стартера посылает небольшую электрическую мощность на соленоид стартера, когда вы включаете ключ зажигания, в то время как соленоид потребляет большой ток непосредственно от автомобильных аккумуляторов. Это заставляет соленоид передавать мощность на стартер, который, в свою очередь, раскручивает маховик.

Все современные стартеры следуют этому процессу. Они полагаются на реле стартера, которое передает мощность на соленоид, включая стартер для вращения маховика. Реле стартера играет важную роль в запуске вашего автомобиля.

Признаки неисправного реле стартера

Как и любой другой механический и электрический компонент в вашем автомобиле, реле стартера показывает некоторые признаки, указывающие на его выход из строя, прежде чем оно наконец остановится. Некоторые из этих признаков аналогичны симптомам неисправного соленоида стартера, а некоторые указывают на неисправный стартер. Это затрудняет поиск. Давайте посмотрим на эти симптомы ниже.

Машина не заводится

Наиболее частым признаком неисправности или выхода из строя реле стартера является то, что ваш автомобиль не заводится, когда вы пытаетесь его завести.Если вы пытаетесь завести автомобиль, но нет никаких признаков запуска или даже щелчка, а внутреннее освещение ярко, это может указывать на неисправное реле стартера.

Проблема может заключаться в перегорании предохранителя реле стартера автомобиля. Независимо от того, сколько раз вы пытаетесь, машина не заводится. Однако если вы слышите щелкающий звук при попытке завести двигатель, это может означать, что реле стартера не полностью испортилось.

В любом случае вам нужно диагностировать систему зажигания, если вы знаете, что с ней делать.В противном случае обратитесь к профессиональному механику для тщательной проверки.

Реле стартера, которое остается включенным даже после запуска двигателя

Когда вы включаете зажигание, он посылает электрический ток на реле стартера, которое передает ток на соленоид стартера. Соленоид передает мощность на стартер, который вращает гибкую пластину, чтобы запустить двигатель.

При выключении зажигания должно происходить обратное действие; соленоид стартера и двигатель должны перестать работать.Если оно не работает в этой последовательности и реле остается включенным даже после запуска двигателя, у вас неисправное или неисправное реле стартера. Возможно, реле подает устойчивый ток.

Это обычно происходит, когда реле касается чего-либо или подвергается воздействию высокой электрической мощности. Вам необходимо немедленно диагностировать и устранить эту проблему, потому что это может привести к повреждению всей системы стартера.

Серия быстрых нажатий со стартера

Реле стартера щелкает, но отсутствие кривошипа показывает, что стартер не получает достаточного электрического тока от реле для вращения двигателя.Это также признак разряженной или разряженной батареи. Реле работает только тогда, когда оно подает на стартер достаточный электрический ток. Арендодатель высокой мощности может либо повредить всю систему стартера, либо не завести автомобиль, что будет сопровождаться раздражающим щелчком.

Оба могут быть результатом коррозии или старения реле, контактные точки которого повреждены. Ремонт требует только очистки контактных точек для обеспечения надлежащего потока или замены старого реле. Вы можете очистить проржавевшее реле, соскоблив ржавую поверхность наждачной бумагой или скребком.Вы можете подумать о замене реле для лучшего выхода.

Периодические проблемы с запуском автомобиля

Иногда двигатель не запускается, если вы не включите и не выключите зажигание пару раз. Это признак неисправного реле стартера, и в основном это вызвано коррозией, грязью, мусором или длительным воздействием тепла. Коррозия и остатки уменьшат прохождение электрического тока в цепи.

Реле стартера — это упрощенный компонент системы зажигания, который не состоит из многих частей и поэтому редко выходит из строя.Но когда это происходит, это связано с проблемами электропроводности.

Как устранить неисправность пускового реле?

Симптомы могут быть связаны с неисправным генератором переменного тока или разряженной или вышедшей из строя аккумуляторной батареей, которую необходимо заменить или заменить. Чтобы определить реальную стоимость проблемы, вам нужно провести несколько тестов на схемах реле стартера.

Не паникуйте! Вы можете провести тест самостоятельно. В следующих нескольких абзацах мы объясним, как проверить реле стартера в упрощенном виде.

Диагностировать реле стартера довольно легко благодаря его расположению. Все, что вам нужно, — это подходящие инструменты и, конечно же, знания.

Необходимые материалы :

  • Отвертки
  • Контрольные лампы
  • Кусок провода для перемычки
  • Наборы ключей и головок (на случай потери).

Проверка реле стартера

Перед началом теста возьмите полностью заряженный аккумулятор и переносную перемычку.Или вы можете протестировать автомобильный аккумулятор и убедиться, что он хорошо заряжен и не является причиной вашей проблемы. Будьте осторожны с процессом тестирования и помните, как вы размещаете перемычки на клеммах аккумулятора.

Процедуры тестирования

Найдите предохранитель реле стартера. Расположение предохранителя может отличаться от автомобиля к автомобилю. Предохранители в основном расположены рядом с аккумулятором, к которому подсоединен положительный полюс аккумулятора.

Попросите помощника помочь вам повернуть ключ зажигания в положение ON.Если вы слышите слабый щелчок, вам необходимо провести тест на электрическое сопротивление. Но если вы слышите слышимый щелчок, вам необходимо провести диагностику реле стартера на предмет падения напряжения.

Диагностика электрического сопротивления

1). Возьмите контрольную лампу, предпочтительно мультиметр, и установите ее на шкале Ом. Коснитесь одного из щупов на заземляющем проводе, а другим — на клемме цепи зажигания. Хорошее реле стартера должно показывать сопротивление ниже 5 Ом. Любое указанное выше значение указывает на неисправное реле.

2). Второй метод диагностики сопротивления — это подсоединение красного щупа мультиметра к проводу цепи зажигания, а другого щупа к проводу заземления. Если при включении зажигания он показывает менее 12 В, это указывает на неисправное реле.

3). Третий и последний метод проверки электрического сопротивления в нашем списке — использование переносного соединительного кабеля. Подсоедините провод аккумуляторной батареи и провод цепи зажигания. Сильный щелчок реле говорит о том, что оно работает нормально.И один или несколько слабых щелчков показывают, что у вас неисправное реле, которое требует замены.

Диагностика падения напряжения

1). Перезагрузите мультиметр, чтобы он был на 20 В постоянного тока

2). Подключитесь к красному щупу мультиметра на красном выводе аккумуляторной батареи. Поместите черный и тонкий провод на провод, идущий к выключателю цепи зажигания.

3). Попросите помощника повернуть ключ зажигания в положение ON, пока вы проверяете показания мультиметра.Напряжение не должно быть выше 0,2 В. Если мультиметр показывает выше 0,2 В, либо у вас неисправное реле стартера, либо проблема с электропроводностью реле стартера, которая требует должного внимания. Приходится проверять разъемы и чистить их.

Что вызывает отказ реле?

Несколько факторов могут привести к отказу автомобильного реле, в том числе:

Коррозия : Корродированные провода или разъемы не могут передавать необходимое количество тока. Это приведет к проблемам с запуском.Это может быть прерывистый запуск, щелчок реле или неработающий автомобиль, который не запускается.

Грязь и мусор: этот компонент обычно защищают от грязи, мусора и царапин, помещая их под приборную панель или капот. Со временем мусор и грязь будут накапливаться, что отрицательно скажется на работоспособности реле. Поскольку грязь скапливается, она не пропускает адекватный ток, и вы услышите серию жужжащих или щелкающих звуков от реле.

Плохие разъемы : Неисправный или сломанный разъем — важная причина выхода из строя реле.Реле не будет работать должным образом, если не проходит необходимое количество тока. Он будет отображать некоторые признаки, издавая серию слабых щелкающих звуков и не в состоянии нести пусковые цепи. Помимо серии слабых щелчков, автомобиль может отказываться заводиться.

Слишком много тепла : Если через реле постоянно проходит высокое напряжение, оно будет выделять избыточное тепло, которое сожжет или оплавит контакты, заставляя их слипаться. Это замкнет цепь стартера даже при выключенном зажигании.Этот эффект требует немедленного внимания, поскольку может привести к повреждению всей системы запуска.

Слишком старое реле: проблемы с реле стартера могут не иметь существенной причины. Это может быть слишком старая эстафета, которая продержалась сотни миль. У старого реле может быть изношенная деталь, которая не может работать должным образом. В таком случае нужно заменить реле, не более того.

Теперь, когда вам известны симптомы и причины неисправного реле стартера, как их исправить? Ответ зависит от причины сбоя.Если это вызвано грязью или мусором, их очистка будет лучшим вариантом. А если это вызвано перегревом или неисправными разъемами, подумайте о их замене. Это подводит нас к тому, как исправить неисправные проблемы реле стартера.

Как исправить неисправность реле стартера

Если реле вышло из строя из-за коррозии или грязи на выводах, очистка разъемов восстановит реле. Возможно, вам потребуется осмотреть и очистить внутреннюю конструкцию блока реле. В зависимости от того, насколько загрязнены или корродированы разъемы, вам может потребоваться только очистить внешние детали.Для очистки грязи и царапин требуется металлическая щетка и разрыхлитель. Для очистки вам также может понадобиться груша и полотенце из микрофибры.

Независимо от причины, лучший вариант решения проблемы реле стартера — замена реле. Замена реле — это решение проблемы раз и навсегда. Реле стартера — это долговечный компонент, который может прослужить до 100 000 миль. Так что замена стоит затрат. Давайте изучим задействованные процессы.

Замена неисправного пускового реле

Замена реле стартера — довольно простое и быстрое решение.При установке нового реле стартера необходимо соблюдать несколько процессов. Все зависит от типа реле, которое вы заменяете.

Как безопасно снять реле стартера

Блок предохранителей, реле стартера

Шаг 1 : Поднимите капот и отсоедините отрицательный полюс аккумуляторной батареи с помощью большого гаечного ключа.

Шаг 2: Найдите блок предохранителей. Обычно это коробка с черной крышкой, расположенная рядом с аккумулятором. Если вам сложно его идентифицировать, обратитесь к буклету владельца.Буклет поможет вам найти реле стартера.

Шаг 3: Отсоедините реле.

Настенное реле Fender

Это реле стартера монтируется непосредственно на брандмауэре или крыле. Чтобы удалить его, выполните следующие действия.

Шаг 1: Отсоедините клемму аккумулятора с помощью большого гаечного ключа.

Шаг 2: Отсоедините провода, идущие к клеммам реле. Используйте соответствующий гаечный ключ или головку, чтобы ослабить гайки, удерживающие выводы реле.

Шаг 3: Снимите крепежные винты, которыми реле крепится к крылу.

Установка реле стартера

Установка реле стартера блока предохранителей — одна из самых простых работ своими руками. Нет болтов, винтов или слишком затянутых гаек — и не о чем беспокоиться. Сравните новое и старое реле. Убедившись, что они такие же, возьмите новое реле и совместите контакты в их гнездах в блоке предохранителей. Осторожно надавите на него, пока он не дойдет до конца паза и не сядет равномерно. Закройте блок предохранителей и снова подсоедините клеммы аккумулятора.

Выполните следующие шаги, чтобы установить настенное реле на крыльях;

Приложите реле к месту установки и удерживайте его.Вставьте винты и затяните их, чтобы удерживать реле. Подключите выводы цепи стартера к клеммам аккумуляторной батареи. Убедитесь, что вы подключили правильный провод к правильной стойке.

После установки убедитесь, что сомневаетесь, и перепроверьте работу. Убедитесь, что кабели и разъемы на настенном реле крыла не повреждены. Протестируйте стартовую систему. Если все работает, похлопайте себя по плечу. Однако, если вы не любитель домашних мастеров и вас не устраивает это руководство, не стесняйтесь обращаться за профессиональной помощью.

Часто задаваемые вопросы:

Щелкает ли реле в плохом состоянии?

Если у вас неисправное или неисправное реле стартера, оно не будет обеспечивать достаточный электрический ток для питания стартера. В результате, независимо от того, сколько раз вы проворачиваете двигатель, он не запускается. Неисправное реле стартера издает щелчок при включении зажигания.

Где находится выключатель реле стартера?

Реле стартера обычно устанавливаются в моторном отсеке, но не на блоке двигателя.Вы можете отследить реле стартера блока предохранителей, проследив за большим проводом от положительной клеммы аккумуляторной батареи. Они расположены рядом с аккумулятором в ящике с черной крышкой на большинстве автомобилей.

Как проверить, работает ли реле стартера?

Единственный инструмент, необходимый для проверки реле стартера, — это мультиметр. Возьмите мультиметр и установите его по шкале Ом. Поместите один из щупов на кабель заземления, а другой щуп на клемму цепи зажигания.

Хорошее реле должно быть меньше 5 Ом.Если показания превышают 5 Ом, это означает, что у вас неисправное реле стартера, которое требует замены.

Последние мысли

Очень важно убедиться, что реле стартера работает правильно. Считайте это частью регулярного технического обслуживания вашего автомобиля. Следовательно, знание симптомов неисправного реле стартера и способов их устранения спасет вас от застревания в глуши.

Я считаю, что эта статья познакомила вас с информацией и процедурами, необходимыми для устранения проблем с реле стартера.Теперь вы знаете функции, причины и признаки реле стартера. Вы можете диагностировать, исправить или заменить неисправное реле стартера.

Подробнее:

Разница между реле стартера и соленоидом

Многие думают, что реле стартера — это то же самое, что и соленоид. Если вы не механик и у вас нет лебедки, это, вероятно, безобидное предположение. Но для владельцев лебедок, любящих бездорожье, это может стать серьезной ошибкой. Лебедка имеет много важных частей, которые делают ее бесполезной, если одна из этих частей повреждена.

Часто ваша лебедка может быть повреждена при движении по бездорожью. Возможно, вам придется отремонтировать лебедку (надеюсь, с вами этого не случится, вот как защитить себя на бездорожье). В такие моменты очень важно не допускать ошибок, предположений и необоснованных предположений.

Итак, давайте узнаем, в чем разница между реле стартера и соленоидом.

Что такое реле стартера?

Первое, что вам нужно знать о реле стартера на лебедке: у вашей лебедки нет реле стартера.

Это может шокировать вас. Это потому, что слова «реле стартера» часто используются как синонимы слова «соленоид». Многие люди и даже веб-сайты говорят, что это одно и то же. Вот почему это может сбивать с толку.

Реле стартера является частью системы запуска автомобиля. Также частью этой системы является соленоид. Вот почему может возникнуть путаница, какая часть есть какая. Кстати, внешне эти две части тоже имеют схожесть. Но функции у них разные.

Что делает реле стартера?

Хотя это всего лишь небольшое электрическое устройство в большой пусковой цепи транспортного средства, реле подает мощный ток на двигатель транспортного средства. Эта функция очень похожа на соленоид лебедки — отсюда и путаница. Но разница в том, что реле стартера — это просто переключатель, в то время как соленоид лебедки является одновременно реле и исполнительным механизмом.

Помимо автомобилей, реле стартера также используется в качестве дистанционного переключателя, который управляет цепями с большим током.К таким приложениям относятся холодильники, мотоциклы, газонокосилки и многое другое. Одна из вещей, которая не нуждается в реле стартера, — это лебедка. И в этом основное различие между ними.

Различия между реле стартера и соленоидом

Вот главное отличие реле стартера от соленоида. Имейте в виду, что обсуждаемый здесь соленоид относится к соленоиду в цепи запуска автомобиля. Как уже объяснялось, у лебедки нет реле стартера.Так что вы не запутаетесь, является ли его соленоид реле.

Что касается дальнейшего обсуждения реле стартера и соленоида на транспортном средстве, давайте просто сделаем это для обучения. Чем больше вы знаете, тем лучше вы подготовлены к уходу за своим автомобилем.

Размер

Первое различие, которое можно заметить в этих двух машинах, когда они размещены рядом друг с другом, — это их размеры.

Реле стартера значительно меньше, чем более мощный соленоид. Это верно независимо от того, идет ли соленоид от лебедки или соленоида стартера автомобиля.

Функции

Внутренняя конструкция соленоида состоит из двух катушек с проволокой (аналогичных катушкам Теслы) и магнитного сердечника на одном конце. Этот сердечник перемещается внутрь и наружу при срабатывании соленоида.

При срабатывании реле стартера через обмотку катушки течет ток. Это создает электромагнитное поле. Это электромагнитное поле перемещает один из видимых плунжеров или якорей, замыкая цепь, тем самым включая двигатель.

В соленоиде, однако, при создании электромагнитного поля движется сердечник.Это движение выполняет две функции: замыкает цепь и перемещает шестерню, чтобы работать как привод.

Размещение

Функция соленоида в системе запуска транспортного средства состоит в том, чтобы активировать двигатель и заставить его работать. Вот почему соленоид обычно находится рядом с корпусом двигателя. Часто соленоид стартера находится рядом с двигателем или на нем.

Что касается реле стартера, то его расположение зависит от типа или модели автомобиля. Может быть в блоке питания или предохранителях; под панелью приборов, внутри панели предохранителей; или это может быть в правом крыле.Конечно, вы также можете найти это под капотом. Реле часто устанавливается на стороне водителя транспортного средства.

Но опять же, расположение реле может быть даже не в любом из упомянутых выше мест. В таком случае обратитесь к руководству по эксплуатации вашего автомобиля.

Требуется ли для лебедки реле стартера?

При установке новой лебедки многие люди, как правило, подключают ее к реле стартера, думая, что двигатель лебедки — это, по сути, переделанный стартер.В этом нет необходимости.

Лебедка не требует подключения к реле стартера. Устанавливая лебедку, вы должны напрямую подключить ее к аккумулятору вашего автомобиля.

Соленоида достаточно, чтобы контролировать ток, протекающий в двигатель вашей лебедки. Если вам нужна помощь в проведении стендовых испытаний всей лебедки на наличие проблем, см. Это пошаговое руководство.

Последнее слово

Опять же, у лебедки нет реле стартера. Сообщая об этом своему специалисту по ремонту лебедки, не сбивайте его с толку, говоря, что у вас сломано реле стартера.

Реле стартера является частью транспортного средства или других машин, которым требуются цепи с большими токами, система запуска. Он работает как переключатель двигателя, тогда как соленоид действует как переключатель и как привод.

Если вам нужна помощь в установке новой лебедки, и если вы не знаете, какие детали куда идут, вот основное руководство.

Соленоид реле стартера — 1PZ® Марка

Все продукты Все products110cc11hp150 GT150 JR150cc150GTS150GTSS150SS182800-1570188F190F1D0-81940-01-001D0-81940-02-001J7-81940-10-001J7-81940-60-00200520072008200cc250cc27800051327800137627800164127800176627800180227800234727800301231204-ZA0-00331800-30B0031800-3340035035850-HA8-77135850-HF1-6704-stroke49cc4BH-81940 -004BH-81940-014BH-81940-024BH-81940-02-004JG-81940-124KB-81940-01-004KD-81940-00-004SV-81940-00-004XE-81410-11-004XE-81940-00-004XE -81940-10-004XE-81940-12-005GT-81940-00-005UG-81940-00-00800 VL800800 VL800Z Limited9hpAfronAIM-EXAIMEXAllmandArctic 300Arctic CatATCATC125MATC200ATC250ATC250ESATP 330ATABBearBear Traig 250BearBear Traig 250BearBear Traig 250Bear 250BMXBruinBruin 250BuyangCA175CARROLLCB 360CB 400CB 450CB175CB200TCB350CB350FCB350F FourCB360TCB400CB400FCB400T HawkCB450CB500CB500TCB550KCB750CB750A HondamaticCh225Ch540 EngineCL175CL200CL350CL450CM 450 ECM200CM250CCM400CCM450ACOOLSPORTCX500CDIRT BIKEEAGLEFJR13 00FJR1300A ABSFJR1300AEFL350R PilotFL400RFL400R PilotFM 660 RForemanForeman 400Foreman 450Foreman 500Foreman Рубикон 500Foreman400FourTraxFourtrax 125FourTrax 200Fourtrax 250Fourtrax 300Fourtrax Ринкон 650FZ1FZ1 Команда YamahaGIOVANNIGL1800go kartGo-KartGOLDWINGGoldwing 1000GOLDWING 1900GrizzlyGrizzly 350Grizzly 450Grizzly 550Grizzly 600Grizzly 660GSX-R750GTIGTXGXV240 8hpGXV270GXV270 9hpGXV340GXV340K1GXV390GXV390 13hpGY6Hammerhead 150GTHAMMERHEAD 150SSHammerhead 250GTHAMMERHEAD TWISTERHawk 400hodnahodna TRX300FWHondaHonda CB750KHonda CL450HONDA CLONESHonda GL 1000honda250HP1-601JACKELJCLJet BoatJMSTARJONWAYKawasakiKawasaki KZ 1000Kawasaki Z1KazumaKING QUAD 300KLT200KodiakKodiak 2000Kodiak 350Kodiak 400KOHLERKYMCOKZ750LT-F250TLT160ELT230ELT230FELT4WDLTF160LTF160ELTF250LTF250 2x4LTF250FLTF300FMagnum 325Magnum 425Magnum 500Magnum330Maxim 650Maxim 750MopedMotorMotorcycleMotorsportNSTPANTERRAPEACEPhoenix 200PIONEERPioneer 700Pioneer700Pioner 700PIT BIKEpocket bikePolarisPolaris 300Polaris 400L Polaris ATVQLINKQMB139QuadQuadrunner 160RaiderRaider пуля CowlRaider XVRANCHERRancher 350Rancher 420Rancher350Rancher400Rancher420Ranger 400Ranger 500Ranger 700Ranger 800Ranger XP 900RaptorRaptor 350Raptor 660Raptor 700Raptor 700RRecon 250RED CATRedcatRhinoRhino 450Rhino 660Rhino 700RINCONRincon 680Rincon680Road StarRoadlinerROKETARoyal StarRubiconRubicon 500RXPRXTRZR 570ScooterScootersScramblerScrambler 400Scrambler 500Sea-DooSea-Ду 3DSea-Ду GTISea-Ду GTRSea-Ду GTSSea-Ду GTXSEASENGSilveradoSL350SnowmobilessolenoidSolenoid Starter RelaySOLENOID SWITCHSPORTRAXSportrax 250Sportrax 300Sportrax 400Sportrax2006Sportrax90Sportsman 335Sportsman 400Sportsman 500Sportsman 600Sportsman 700SRX250starter relayStarter реле SolenoidStarter реле соленоида соленоида SwitchStarter relaystarter зр; empod relaySTREAMSUN-LSunlSUZUKISuzuki BoulevardSuzuki BurgmanSuzuki Burgman 400Suzuki GSXR1000Suzuki GSXR1000 ABSSuzuki GSXR1000RSuzuki GSXR600Suzuki GSXR750Suzuki GZ250Suzuki Интрудер 1500Suzuki InTru дер 1500 VL1500Suzuki Intruder 1500 VL1500B BlackSuzuki Intruder Volusia 800Suzuki Katana 600Suzuki Katana 600 GSX600FSuzuki Katana 750Suzuki Katana 750 GSX750FSuzuki King Quad 300Suzuki Marauder 800Suzuki Marauder 800 VZ800Suzuki Озакр 250Suzuki Quadsport Z250Suzuki RF900RSuzuki SV1000Suzuki SV1000SSuzuki SV650Suzuki SV650ASuzuki SV650SSuzuki SV650SFSuzuki TL1000RSuzuki TL1000SSuzuki TU250XSuzuki VanVan 200Suzuki Vstrom 1000Suzuki Vstrom 650SXS700SXS700M2TANKTaoTaoTimberwolfTimberwolf 250ToroTouring 175Trail Blazer 250Trail Boss Boss 250Trail 350Ltrx 350TRX 500TRX 90 XTRX125trx200TRX200 SXTRX200DTRX200SXTRX250TRX250EXTRX250RTRX250TETRX250TE 2×4 ESTRX250TMTRX250TM 2x4TRX250XTRX300TRX300EXTRX300FWtrx350TRX350DTRX350FETRX350FMTRX350TETRX350TMtrx400TRX400 FWTRX400EXTRX400FWtrx420TRX420FATRX420FETRX420FPATRX420PETRX420TETRX420TMtrx450TRX450ERTRX450ESTRX450RTRX450Strx500TRX500FATRX500FE2TRX500FGATRX500PGTRX650TRX650FAtrx680TRX680FATRX700TRX90TRX90XTW200UTVV Star 1100V Star 650variousVentoVE SPAViking 700VIPVirago 500Virago 535Virago 535SVTX1800CVTX1800C2VTX1800F1VTX1800F2VTX1800F3VTX1800N1VTX1800N3VTX1800R1VTX1800R2VTX1800R3VTX1800SVTX1800S3Warrior 350warrior 350YFM350WolverineWolverine 350WR 250FWR250X1X18X19X7XJ550XJ550RXJ650XJ650LXJ650RXJ750XJ750MXJ750RXJ900RXplorer 30Xpress 300XS360XS400XS400RXS750XS750SXT225XTREMEXV1100XV1100SXV535XV750XV920XV920RXVS1100XVS1100 CustomXVS1100AT SilveradoXVS650yamahaYamaha Максим 650Yamaha Максим 750Yamaha Seca 550Yamaha Seca Turbo 650Yamaha Virago 1100yamaha 400Yamaha ATVYamaha Badger 80Yamaha Big BearYamaha Big Bear 350Yamaha Kodiak 450Yamaha Максим 400Yamaha Максим 550Yamaha Мото-4Yamaha Raptor 700Yamaha Raptor 700RYamaha Rhino 450Yamaha Rhino 660Yamaha Rhino 700, Yamaha Road Star, Yamaha Seca 550, Yamaha Seca 900, Yamaha SR 250, Yamaha SRX250T, Yamaha SRX250TC, Yamaha Timberwolf 250, Yamaha Trailway, Yamaha Trailway 200, Yamaha TTR 225, Yamaha V Star, Yamaha Viking 700, Yamaha Viking, VIYamaha Virago, Wirago, 535, Yamaha, Viking, Virago, Virago, 535, Yamaha, Viking, Yamaha, Wirago, 535, Yamaha. Rine 350Yamaha Росомаха Р-SpecYamaha Росомаха Росомаха X2Yamaha X4Yamaha XJ550Yamaha XJ650Yamaha XJ750Yamaha XS XS 360Yamaha 400RKYamaha XS360Yamaha XS400Yamaha XT 225Yamaha XV 1700Yamaha YFA 125yamaha YFM40Yamaha YX600 RadianYFA 125YFB 250YFB250YFB250FWYFMYFM 100YFM 225YFM 250YFM 350YFM 400YFM 450YFM 660YFM 660 RYFM 700 RYFM225YFM250YFM250XHyfm350YFM350FGYFM350FHYFM350FWYFM350FXYFM350UYFM350Xyfm400YFM400FBYFM450YFM550YFM600FYFM600FWAYFM660YFM700YFM80YFN 80YFP350yfz450YIBENYXE70YXM700YXM700DHYXM700PYXR 450YXR 660YXR700FDUYXR700FSPYZF R1YZF- R1YZF- R6YZF- R7YZFR1YZFR6

признаков неисправного соленоида стартера — все, что вам нужно знать

Электромагнит стартера отвечает за передачу электрического тока от аккумулятора к самому стартеру.

Авторемонт стоит ДОРОГОЙ


Хотя неисправность соленоида стартера не является очень распространенной, он может быть поврежден из-за чрезмерного нагрева, влажности или плохой проводки.

К счастью, соленоид стартера предупредит вас, когда он вот-вот выйдет из строя, еще до того, как это произойдет.

В этой статье рассматриваются наиболее распространенные признаки неисправного соленоида стартера, а также общие причины повреждения соленоида. Мы также предоставим вам пошаговое руководство по самостоятельной замене соленоида стартера, чтобы сэкономить на трудозатратах.

Что такое соленоид стартера?

Соленоид стартера — один из самых забытых и важных компонентов системы зажигания.

Электромагнит стартера отвечает за получение энергии от батареи реле стартера и активацию стартера для запуска двигателя. Таким образом, без реле стартера электрическая цепь не замкнута, и двигатель не может запуститься.

Помимо передачи тока аккумуляторной батареи, соленоид стартера отвечает за создание электромагнитной силы для создания требуемой механической силы, необходимой для перемещения шестерни.Как только шестерня движется, запускается маховик двигателя.

Соленоид стартера сделан очень прочным, и очень редко приходится иметь дело с неисправным соленоидом стартера. При этом соленоид стартера не работает вечно, и вы можете столкнуться с ситуациями, когда соленоид стартера неисправен.

Каковы основные признаки неисправного соленоида стартера?

Как и любой другой компонент вашего автомобиля, соленоид стартера сообщает вам, когда он выходит из строя, еще до того, как это произойдет.

Есть очень частые признаки неисправного соленоида стартера; некоторые из них легко обнаружить, в то время как другие немного сложнее и требуют некоторого уровня внимательности, прежде чем вы их обнаружите.

  • Соленоид стартера издает быстрый щелчок

Как водитель, вы всегда должны помнить, что любой странный шум, исходящий от вашего автомобиля, не является хорошим признаком, независимо от того, исходит ли он от соленоида стартера или других деталей.

Как только вы услышите эти шумы, вы должны немедленно принять меры и передать автомобиль профессиональному механику для проверки и ремонта. Хотя некоторые из этих звуков могут возникать из-за простых проблем, другие звуки могут быть очень серьезными и могут указывать на значительный ущерб.

Один из первых и наиболее распространенных признаков неисправного соленоида стартера — это странные быстрые щелкающие звуки. Эти звуки происходят из-за того, что соленоид не получает нужного количества электрического тока.

Одна из основных причин этой проблемы — слабые соединения и проблемы с проводкой.

  • Стартер продолжает вращаться

При отпускании кнопки стартера или выключении зажигания стартер должен перестать вращаться. Однако с неисправным соленоидом стартера стартер не поймет, как вы выключите переключатель и продолжите движение.

По мнению автомобильных экспертов, одной из первых причин непрерывного вращения стартера является оплавление или повреждение поверхности соленоида стартера из-за сильного нагрева.

Об этой проблеме необходимо позаботиться, чтобы избежать серьезных повреждений, требующих очень больших затрат на ремонт.

  • Щелчок соленоида без вращения стартера

Когда вы включаете ключ в корпусе ключа, соленоид стартера перемещается, затем стартер вращается, чтобы запустить двигатель.

Когда ваш соленоид стартера выходит из строя, вы можете услышать щелкающий звук и небольшое движение в соленоиде стартера, но вы не увидите соответствующего вращения стартера, и, следовательно, двигатель не запустится.

В этом случае виновником может быть обрыв соединения соленоида из-за эрозии, поломки или грязи.

  • Ведущая шестерня заднего хода

обычно, когда стартер начинает вращение, ведущая шестерня не должна двигаться назад; в противном случае неисправен соленоид стартера.

Одной из самых серьезных проблем с корнем в этом случае может быть сломанная пружина из-за потери электричества.

Какие типичные примеры неисправностей соленоида стартера?

Как мы упоминали ранее, неисправный соленоид стартера встречается очень редко. Однако, если соленоид стартера выходит из строя по какой-либо причине, вот что действительно могло пойти не так:

  • Не удается сбросить соленоид стартера

Когда соленоид стартера выходит из строя, очень вероятно, что эта деталь застрянет и не сбросится.Даже если вы попытаетесь выключить зажигание или отпустить кнопку стартера, соленоид стартера не сможет вернуться в исходное положение.

  • Электромагнит стартера не работает как обычно

Еще одним важным примером неисправного соленоида стартера является то, что он теряет способность частого всасывания.

  • Ведущая шестерня не восстанавливается в нужный момент

Когда соленоид стартера выходит из строя, возвратная пружина может становиться все слабее и слабее, что приводит к обратному действию коронной шестерни маховика двигателя.Это обратное действие обычно происходит, когда ведущая шестерня не восстанавливается в нужное время.

  • Соленоид стартера не может запустить стартер

Наконец, когда соленоид стартера выходит из строя, он больше не сможет запустить стартер. Вместо этого стартер будет издавать непрерывные звуки, пока двигатель не работает.

Что вызывает неисправный соленоид стартера?

Причин, приводящих к неисправному соленоиду стартера, множество, в том числе:

Очень часто приходится иметь дело с ослабленными или неправильно подключенными проводами в системе стартера вашего автомобиля.

Поскольку эти провода выходят из строя, соленоид стартера не получает нужного количества тока, что приводит к его повреждению.

  • Проблемы, связанные с тепловыделением

Соленоид стартера не предназначен для выдерживания высоких токов в течение более длительных периодов времени, потому что большой ток генерирует много тепла, которое может повредить соленоид.

Если оставить ключ зажигания в положении «включено» на долгое время, соленоид стартера может выйти из строя из-за длительного воздействия сильного тока.

  • Проблемы со слишком большим количеством влаги

Как и большинство деталей, связанных с электричеством, вы никогда не хотите, чтобы на стартер попадала влага, чтобы продлить срок его службы.

Если влага попала в соленоид стартера, вы заметите щелчок, после которого стартер не запустится.

Эта влага вызывает коррозию соленоида, что приводит к значительному снижению эффективности соленоида.

Соленоид стартера — один из тех компонентов автомобиля, которые не нужно перетягивать ни по какой причине.

При чрезмерной затяжке болтов соленоида стартера существует высокая вероятность повреждения или изгиба этих болтов внутри самого соленоида стартера. В результате электрический ток и, следовательно, эффективность соленоида стартера значительно снижаются.

Что делать, если неисправен соленоид стартера?

Предположим, вы заметили какой-либо из ранее упомянутых признаков неисправного соленоида стартера.В этом случае вы должны отвезти свой автомобиль к профессиональному механику, осмотреть его и позаботиться о любом ремонте.

Некоторые из этих ремонтов могут быть такими же простыми, как замена некоторых систем проводки, в то время как другие ремонты могут потребовать замены всей части соленоида стартера.

Перед тем, как принимать окончательные решения по ремонту, вы должны сначала подтвердить механику, что соленоид стартера является виновником, потому что другие компоненты могут повлиять на работу соленоида стартера в системе запуска.

Как устранить неисправность соленоида стартера?

Важно убедиться, что проблема запуска связана с соленоидом стартера, прежде чем предпринимать какие-либо действия и заменять его.

Вот что вам нужно сделать для устранения неисправности соленоида стартера:

  • Не отключая аккумулятор, включите аварийный тормоз и поставьте автомобиль на нейтраль или припаркуйте
  • С помощью вольтметра осмотрите аккумулятор и убедитесь, что с самим аккумулятором все в порядке
  • Вернитесь на место водителя и включите зажигание, наблюдая за освещением приборной панели.
  • Проверьте напряжение в проводах среднего размера в батарее и убедитесь, что в проводе среднего размера около 12 Вольт.
  • Пожалуйста, выньте красный порт, подключенный к соленоиду стартера, и подключите его к перемычке и следите за искрой
  • Попросите помощника включить переключатель и прислушаться к звуку, указывающему на включение стартера.
  • Если вольтметр не показывает 12 вольт и стартер не активируется, значит, проблема с соленоидом стартера, и его необходимо заменить.

Как заменить соленоид стартера?

Если вы подтвердили, что соленоид стартера является виновником вашей системы стартера, соленоид необходимо заменить.

Поскольку часть соленоида стартера не очень дорогая, возможно, стоит узнать, как заменить соленоид стартера самостоятельно, вместо того, чтобы тратить деньги на оплату труда.

Вот пошаговое руководство по замене соленоида стартера вашего автомобиля:

  • Поднимите автомобиль домкратом и подставкой для домкрата. Если вы никогда раньше не поднимали свой автомобиль, важно обратиться к руководству по эксплуатации автомобиля для получения более подробных инструкций. Если у вас нет экземпляра руководства, вы можете загрузить его из Интернета или получить копию у местного механика.
  • Когда автомобиль будет поднят, отсоедините кабели аккумуляторной батареи, чтобы предотвратить поражение электрическим током при работе с автомобилем.
  • Найдите стартер вашего автомобиля под автомобилем. Если вы не можете найти его визуально, вы можете обратиться к руководству пользователя для получения более подробной информации.
  • Перед тем, как отсоединить стартер, обязательно сделайте заметку или сделайте снимок и отметьте, какой провод с какой стороны подключен. Один из проводов должен быть подключен к аккумулятору, и вы не захотите переключать эти провода при переустановке стартера.
  • Отсоедините стартер с помощью гаечного ключа и отвертки и, если требуется, выньте крепежные болты стартера
  • Теперь отсоедините соленоид стартера от самого стартера, открутив два больших болта
  • Сравните старый соленоид с новым, чтобы убедиться, что вы выбрали правильный.Вы также можете обратиться к руководству по эксплуатации транспортного средства для более подробного описания того, какой соленоид вам следует купить.
  • Установите новый соленоид на место, сначала надев его на стартер, а затем осторожно прикрутив.
  • Установите стартер на свой автомобиль и обязательно следуйте инструкциям при соединении двух проводов.
  • Проведите тест-драйв своего автомобиля и убедитесь, что все работает правильно. Имейте в виду, что если автомобиль не заводится, проблема может быть не связана с неправильной установкой; это может быть другой компонент в системе стартера, вызывающий проблему.

Иногда проблема запуска может быть связана с самим стартером, а не только с соленоидом стартера. В этом случае, к сожалению, вам нужно заменить весь стартер примерно за 430-700 долларов.

Перед заменой стартера нужно поставить обратно и убедиться, что стоит замена. Другими словами, если ваш автомобиль очень старый, а затраты на ремонт приближаются к стоимости автомобиля, вы можете подумать о том, чтобы бросить его на свалку.

К счастью, покупатель Cash Cars платит больше всего денег за утиль вместе с БЕСПЛАТНОЙ буксировкой! Позвоните нам, чтобы узнать подробности.

Заключение

Соленоид стартера отвечает за передачу электрического тока от аккумулятора к стартеру для запуска двигателя.

Хотя неисправный соленоид стартера не является очень распространенным явлением, существуют общие признаки неисправного соленоида стартера, включая частый щелкающий звук соленоида стартера, непрерывное вращение стартера без запуска двигателя, стартер не может вращаться и двигаться. передача реверсирует.

Если вы заметили какой-либо из этих признаков, вы должны сначала осмотреть автомобиль профессиональным механиком, чтобы убедиться, что соленоид стартера вызывает проблему, а во-вторых, чтобы решить проблему.

Неисправный соленоид стартера может вызвать головную боль и привести к серьезным проблемам с автомобилем. Таким образом, вам нужно как можно скорее о ней позаботиться.

Добавить комментарий

Ваш адрес email не будет опубликован. Обязательные поля помечены *